You are on page 1of 34

VISIONIAS

www.visionias.in
ANSWERS & EXPLANATIONS
GENERAL STUDIES (P) TEST – 3185 (2021)

Q 1.A
• Due to salinity, plants are not able to draw as much water from the soil. When the soil salinity levels are
high enough, the water in the roots is pulled back into the soil. The plants are not able to take in enough
water to grow.
• The major source of salinity problems is usually irrigation water.
• Salt-affected soils can be corrected by:
o Improving drainage: In soils with poor drainage, deep tillage can be used to break up the soil surface
as well as claypans and hardpans, which are layers of clay or other hard soils that restrict the
downward flow of water. Tilling helps the water move downward through the soil. Hence option 1 is
correct.
o Leaching: Leaching can be used to reduce the salts in soils. Adding enough low-salt water to the soil
surface to dissolve the salts and move them below the root zone. The water must be relatively free of
salts (1,500 – 2,000 ppm total salts), particularly sodium salts. A water test can determine the level of
salts in your water. Hence option 2 is correct.
o Reducing evaporation: Applying residue or mulch to the soil can help lower evaporation rates. As an
increase in evaporation increases salinity. Hence option 3 is not correct.
o Applying chemical treatments: Before leaching saline-sodic and sodic soils, treating them with
chemicals, to reduce the exchangeable sodium content reduces salinity. Gypsum is the most
commonly used to correct saline-sodic or sodic soils. Hence option 4 is correct.
Q 2.C
• Earth is currently experiencing a biodiversity crisis. Recent estimates suggest that extinction threatens up
to a million species of plants and animals, in large part because of human activities such as deforestation,
hunting, and overfishing. Today, extinctions are occurring hundreds of times faster than they would
naturally. The previous five mass extinction episodes are:
o Ordovician-Silurian extinction - 444 million years ago
o Late Devonian extinction - 383-359 million years ago
o Permian-Triassic Extinction - 250 million years ago.
o Triassic-Jurassic Extinction: 210 million years ago.
o Cretaceous-tertiary Extinction: 65 Million Years Ago - The Cretaceous-Paleogene extinction event
is the most recent mass extinction and the only one definitively connected to a major asteroid impact.
Some 76 per cent of all species on the planet, including all nonavian dinosaurs, went extinct during
this period.
• Statement 1 is correct: During the long period (> 3 billion years) since the origin and diversification of
life on earth, there were five episodes of mass extinction of species. The ‘Sixth Extinction’ presently in
progress is different from the previous episodes in the rates; the current species extinction rates are
estimated to be 100 to 1,000 times faster than in the pre-human times and our activities are
responsible for the faster rates. Ecologists warn that if the present trends continue, nearly half of all the
species on earth might be wiped out within the next 100 years.
• Statement 2 is correct: Some examples of recent extinctions include the dodo (Mauritius),
quagga (Africa), thylacine (Australia), Steller’s Sea Cow (Russia) and three subspecies (Bali, Javan,
Caspian) of the tiger. The last twenty years alone have witnessed the disappearance of 27 species.
Q 3.D
• Recently, the Ministry of Jal Shakti has launched a new version of the India Water Resources
Information System (India-WRIS) with new functionalities and features.
• India-WRIS WebGIS aims as a ‘Single Window’ solution for comprehensive, authoritative and
consistent data & information of India’s water resources along with allied natural resources in
1 www.visionias.in ©Vision IAS
a standardized national GIS framework. The project has been jointly undertaken by CWC, Ministry of
Jal Shakti and NRSC, ISRO, DoS in year 2009.
• Based on the requirements and data availability, the India-WRIS Web-GIS has 12 major info systems, 36
sub info systems including 95 layers, classified under 5 major groups :
o Watershed Atlas
o Administrative Layers
o Water Resources Projects
o Thematic Layers
o Environmental Data
• The basis for all hydro-met observation data is the Water Information Management System (WIMS).
Through a secured login, Central and State Water Agencies can enter, analyse, validate and manage
data for water level (both surface water and ground water), flow, water quality, sediment and many
climate parameters. Hence statement 2 is correct.
• Through this portal, any stakeholder can visualize the information in a user friendly manner, as well
download the information in the form of excel reports and graphs. The water information is easily made
available to users and general public, for decision makers. Hence statement 3 is correct.
• Open to the public, and accessible through the web portal www.indiawris.gov.in, this portal contains
information related to Water Resources through dashboards for rainfall, water levels & discharge of
rivers, water bodies, groundwater levels, reservoir storages, evapotranspiration and soil moisture, as well
as modules on water resources projects, water bodies, hydro-met data availability and tools for GIS
layer editing. Hence statement 1 is correct.

Q 4.C
• The acid rain affects living and non- living components of environment.
• Impact of Acid Rain on Monuments: Acid is corrosive by nature. Hence, water containing acids will
also be naturally corrosive. Even dry acids deposited in air causes damage to limestone, marble and
metals. Acid rain causes heavy damage to monuments and other buildings containing limestone, marble
and metals as acids quickly react with these substances. The damage caused to rocks and marble by
acid rain is called as marble-leprosy or stone-leprosy. In India acid rain was reported to cause
damage to the walls of Taj Mahal in Agra. Hence option (c) is the correct answer.
• Impacts on Human Health: Acids of rain water join food-chains and water cycles and reach to human
body systems. There they cause various types of health problems like neurological and digestive
disorders, problems of eyes, throat and respiratory tract. Acid rain causes varieties of safety hazards like
reduced visibility due to smog etc.
• Impact on Flora and Fauna: Acid rain has seriously adverse impacts on aquatic as well as terrestrial
flora and fauna. It kills fishes in lakes and ponds. Sweden and U.S.A. have 15000 and 100 fishless lakes
respectively due to acid rains.
• Acid of acid rains reaches to wild animals through water cycle and food- chains and cause serious health
problems often leading to death. Acidic water containing traces of toxic metals destroys root systems of
plants.

Q 5.D
• A sacred grove or sacred woods are any grove of trees that are of special religious importance to a
particular culture. Sacred groves feature in various cultures throughout the world. In India, the
introduction of Wild Life (Protection) Amendment Act, 2002, provides government protection to these
lands.
• A strong concentration of these groves is found in Himachal Pradesh and Kerala. Other places where one
can find sacred groves are Rajasthan, Bihar, Meghalaya and Maharashtra.
o Himachal Pradesh: The state with the highest number of sacred groves, Himachal Pradesh is
meticulous and extremely careful about these natural settings. Shipin is believed to be the largest
deodar grove and contains trees that are thousands of years old.
o Assam: Forest-dwelling tribes such as Bodo and Rabha inhabiting the plains and foothills of Western
Assam have the tradition of maintaining sacred groves which are locally called “Than”.
o Maharashtra: The groves in Maharashtra are called deorais or devrais and are concentrated in
Pune, Ratnagiri, Raigad and Kolhapur. A rare variety of biodiversity prevails in these areas. The
entire state contains approximately 250 groves.
o Kerala: A state known for its stunning natural beauty, Kerala is home to 240 sacred groves known
as kavus. These areas contain more than 3000 rare species of plants. Spread over more than 20
hectares, the largest grove is in Ernakulam.
2 www.visionias.in ©Vision IAS
o Karnataka: Devara kadu is the sacred grove in Karnataka. The district of Kodagu has
approximately 346 ‘devara kadu’s.
o Bihar: Known as sarnas, the groves in Bihar are fairly small and only consist of 2 to 20 trees. Full of
creepers, shrubs and grasses, these trees are generally collected in the Chotanagpur region.
o Rajasthan: These regions in Rajasthan take up various names- oraans of Jaisalmer, Jodhpur, Bikaner,
kenkris of Ajmer, vanis of Mewar, shamlet dehs of Alwar. The largest area is covered by oraans that
provide a different set of biodiversity in distinctive areas.
o Meghalaya: Home to some of the richest groves in the country, Khasi hills in Meghalaya have one
grove called law kyntangs in almost every village. Two of the largest ones are Mawphlang and
Mausmai. The popular myth surrounding the forests dictate that anyone damaging the plants and trees
of the grove will be killed by the forest spirit. This myth has kept people from destroying the area and
thus, it is one of the best preserved natural areas.
• Hence option (d) is the correct answer.

Q 6.A
• An ecosystem is a community of living organisms in conjunction with the nonliving components of
their environment, interacting as a system. These biotic and abiotic components are linked together
through nutrient cycles and energy flows. The Ecosystems are functionally independent units of
nature which show characteristic energy flow and nutrient cycling. Hence option (a) is the correct
answer.
• An ecological niche is the role and position a species has in its environment; how it meets its needs for
food and shelter, how it survives, and how it reproduces. A species' niche includes all of its interactions
with the biotic and abiotic factors of its environment.
• Ecocline: A gradation from one ecosystem to another when there is no sharp boundary between the two. It
is the joint expression of the associated community (coenocline) and complex environmental gradients.
• An ecotone is a transition area between two biological communities, where two communities meet and
integrate. It may be narrow or wide, and it may be local (the zone between a field and forest) or regional
(the transition between forest and grassland ecosystems).

Q 7.A
• In April, 2020, as a part of the National Policy on Electronics, the Ministry of Electronics and
Information Technology’s (MeitY) notified production linked incentive (PLI) scheme to boost
domestic manufacturing and attract large investments in mobile phone manufacturing and specified
electronic components, including Assembly, Testing, Marking and Packaging (ATMP) units.
• The scheme shall extend an incentive of 4% to 6% on incremental sales (over base year) of goods
manufactured in India for a period of five years subsequent to the base year as defined. Hence option
(a) is the correct answer.
• All electronic manufacturing companies which are either Indian or have a registered unit in India will be
eligible to apply for the scheme.
• The scheme, is expected on one hand to attract big foreign investment in the sector, while also to
encourage domestic mobile phone makers to expand their units and presence in India.
Q 8.B
• The total amount or weight, or energy content, of (a portion of) organisms existing in a specific area
at a particular time, is known as standing crop.
• Statement 1 is not correct: Each trophic level has a certain mass of living material at a particular time
called as the standing crop. A standing crop is the total dried biomass of the living organisms present in a
given environment. It is not limited to the food grains.
• Statement 2 is correct: The standing crop is measured as the mass of living organisms (biomass) or
the number in a unit area. The biomass of a species is expressed in terms of fresh or dry weight.

Q 9.D
• Development of environmental statistics plays crucial role in sustainable development. To achieve 2030
Agenda for Sustainable Development, the ambit of environment statistics needs to cover all components
of environment issues, from extent to quality, adaptation and mitigation measures to governance and
regulation, so as to enable formulation of appropriate policies.
• The National Statistical Office (NSO) of the Ministry of Statistics and Programme Implementation
has addressed this requirement in EnviStats-India report. The report highlights the data on various
environmental factors as a means to raise awareness and help focus on the pivotality of environmental
concerns. Hence option (d) is the correct answer.
3 www.visionias.in ©Vision IAS
• Recently NSO has released EnviStats India 2020 report. The key findings of the report are
o The average number of heat wave days increased 82.6%. It was 157 in 2019.
o The Particulate Matter size was the highest in Delhi among the metropolitan cities.
o Andhra Pradesh had the highest number of slum population followed by Chhattisgarh and Madhya
Pradesh.
o Tube well/hand pump remained the primary source of drinking water in rural India.

Q 10.D
• Biological contaminants are or are produced by, living things. There are many sources of these pollutants:
o Pollens, which originate from plants
o Viruses, which are transmitted by people and animals
o Mould, bacteria, which are carried by people, animals, and soil and plant debris.
o Household pets, which are sources of saliva and animal dander (skin flakes).
o Droppings and body parts from cockroaches, rodents and other pests or insects.
o Viruses and bacteria.
o The protein in urine from rats and mice is a potent allergen. When it dries, it can become airborne.
o Animal dander and cat saliva, house dust, mites, parasites etc.
• Hence option (d) is the correct answer.

Q 11.B
• Recent context: The Supreme Court has permitted the Northern Railways to cut over 400 trees in
the Taj Trapezium Zone (TTZ) for construction of an additional rail track between Delhi and Agra
on the condition of mandatory compensatory afforestation.
• The Central Government in the exercise of the powers conferred under the Environment (Protection) Act,
1986 has constituted the Taj Trapezium Zone Pollution (Prevention and Control) authority in 1998. Hence
statement 1 is not correct.
• TTZ is a defined area of 10,400 sq km around the Taj Mahal to protect the monument from
pollution.
• TTZ is so named since it is located around the Taj Mahal and is shaped like a trapezium.
• It comprises monuments including three World Heritage Sites the Taj Mahal, Agra Fort and
Fatehpur Sikri. Hence statement 2 is correct.

Q 12.A
• Statement 1 is correct and statement 2 is not correct: Organisms occupy a place in the natural
surroundings or in a community according to their feeding relationship with other organisms. Based on
the source of their nutrition or food, organisms occupy a specific place in the food chain that is known as
their trophic level. Producers belong to the first trophic level, herbivores (primary consumer) to
the second and carnivores (secondary consumer) to the third.
• The important point to note is that the amount of energy decreases at successive trophic levels. When an
organism dies it is converted to detritus or dead biomass that serves as an energy source for
decomposers. Organisms at each trophic level depend on those at the lower trophic level for their energy
demands.

4 www.visionias.in ©Vision IAS


Q 13.C
• Recent Context: New rules have been unveiled which allows Aadhaar authentication to ensure better
services for citizens. According to the Aadhaar Authentication for Good Governance (Social Welfare,
Innovation, Knowledge) Rules, 2020, a state or a central government department can request Unique
Identification Authority of India (UIDAI) to allow Aadhaar authentication for "usage of digital platforms
to ensure good governance, prevention of dissipation of social welfare benefits and enablement of
innovation and the spread of knowledge.
• Aadhaar Authentication means the process by which the Aadhaar number along with the demographic
information or biometric information of a Aadhaar number holder is submitted to the Central Identities
Data Repository (CIDR) for its verification and such repository verifies the correctness, or the lack
thereof, on the basis of the information available with it. Hence, statement 1 is correct.
• The Aadhaar number or the authentication thereof shall not, by itself, confer any right of, or be proof of,
citizenship or domicile in respect of an Aadhaar number holder. Hence, statement 3 is not correct.
• The purpose of Aadhaar Authentication is to provide a digital, online identity platform so that the identity
of Aadhaar number holders can be validated instantly anytime, anywhere.
• UIDAI offers Aadhaar-based authentication as a service that can be availed by requesting entities
(government / public and private entities/agencies). However a Supreme Court Judgement in 2016 and
recent Aadhaar Authentication for Good Governance (Social Welfare, Innovation, Knowledge) Rules,
2020 barred private agencies from using Aadhaar database for authentication. Hence, statement 2 is
correct.

Q 14.A
• Forest fires are uncontrolled fires that burn surface vegetation. Forest Fires cause wide-ranging
adverse ecological, economic and social impacts. In a nutshell, forest fires cause following adverse
impacts:
o Loss of valuable timber resources and depletion of carbon sinks. Hence option 1 is not correct
and option 2 is correct.
o Degradation of water catchment areas resulting in loss of water.
o Loss of biodiversity and extinction of plants and animals.
o Loss of wildlife habitat and depletion of wildlife.
o Loss of natural regeneration and reduction in forest cover and production.
o Global warming resulting in rising temperature.
o Loss of carbon sink resource and an increase in the percentage of CO2 in the atmosphere.
o Change in the microclimate of the area making it unhealthy living conditions.
o Soil erosion affecting the productivity of soils and production.
o Ozone layer depletion. Hence option 3 is correct.
o Health problems leading to diseases.
o Indirect effects on agricultural production leading to loss of livelihood.

Q 15.D
• The act of compulsory acquisition of land for different purposes and forcing people to give up their
homes, resource bases, assets and means of livelihood is called as displacement. Large scale
displacement of people occurs due to Mega River Valley Projects, establishment of big industries, mining
activities, development of canal systems, railway constructions and Natural Disasters.
• Displaced people have to settle on a suitable place which may serve as a favourable place for their
living and development. They settle on a new place usually with the intention of making permanent
new settlement. This is called as resettlement.
• For the resettlement of displaced people due to a project, the declaration of an area under the National
Policy on Resettlement and Rehabilitation -2003, Government of India, by the appropriate
Government authority acquired or proposed to be acquired a land for resettlement and rehabilitation of
project affected families is called as a Resettlement Zone.
• Issues Involved in Displacement and Resettlement: Human Rights, governance and accountability,
participation and self determination in development, complexities of resettlement goals, options and
strategies, relevant policy and legal instrument- are some important issues that are involved in
displacement and resettlement.
• According to the report of The World Commission on Dams (March 16, 2000), the rehabilitation
should also incorporate following elements:
o Enhancement of capabilities of people,

5 www.visionias.in ©Vision IAS


o Expansion of social opportunities by addressing the social and personal constraints that restrict
people’s choices
o Landlessness should be converted to land- based settlement
o Joblessness should be substituted with employment
o Provisions of safe nutrition
o Construction of houses
o Improvements in health and wellbeing
o Community reconstructions and social inclusions
• The United Nations Relief and Rehabilitation Administration was created on November 9,
1943. Among services supplied by the UNRRA, the maintenance of refugee camps, vocational
training, orientation for settlement and an extensive tracing service to find lost relatives come on the
priority basis. The Resettlement and Rehabilitation (R&R) of people affected due to natural
calamities involve services like medical, Psychological, preparation for work etc.
• Hence all the options are correct.

Q 16.A
• The Zoological Survey of India (ZSI) was established on 1st July, 1916 to promote survey,
exploration and research leading to the advancement in our knowledge of various aspects of
exceptionally rich wildlife of the erstwhile ´British Indian Empire´. Hence statement 1 is correct.
• The survey has its genesis in the establishment of the Zoological Section of the Indian Museum at
Calcutta in 1875.
• The Zoological Survey of India and the Botanical Survey of India have surveyed more than 70% of the
total geographical area of India for the assessment of total number of species of plants and animals. So
far, about 81,000 species of animals and 49,000 species of plants, have been recorded by these two
institutions, respectively.
• The objectives of ZSI are classified as follows:
• Primary Objectives
o Exploration, Survey, Inventorying and Monitoring of faunal diversity in various States,
Ecosystems and Protected areas of India.
o Taxonomic studies of all faunal components collected.
o Periodic review of the Status of Threatened and Endemic species.
o Preparation of Red Data Book, Fauna of India and Fauna of States.
o Bioecological studies on selected important communities/species.
o Preparation of databases for the recorded species of the country. Join us at Telegram

o Training, Capacity Building and Human Resource Development. Search → Upsc 4 EveryOne
o Maintenance & Development of National Zoological Collections.

o Faunal Identification, Advisory services and Library Services.


o Publication of results including Fauna of India and Fauna of States.
• Secondary Objectives
o Environmental Impact Studies.
o Maintenance and Development of Museum at Headquarters and Regional Stations.
o Development of ENVIS and CITES Centers.
o Research Fellowship, Associateship and Emeritus Scientist Programmes.
o Collaborative research programmes on Biodiversity with other Organizations.
o GIS and Remote Sensing studies for animal diversity as well as for selected threatened species.
o Chromosomal Mapping and DNA finger printing.
• The All India Tiger Estimation done quadrennially is steered by the National Tiger Conservation
Authority with technical backstopping from the Wildlife Institute of India and implemented by
State Forest Departments and partners. Hence statement 2 is not correct.
• Recently, the fourth cycle of the All India Tiger Estimation 2018, results of which were declared to the
nation on Global Tiger Day last year by Prime Minister has entered the Guinness World Record for
being the world’s largest camera trap wildlife survey.

Q 17.A
• Bioremediation refers to the process of using micro-organisms to degrade waste matter. It is a
process used to treat contaminated media, including water, soil and subsurface material, by altering
environmental conditions to stimulate the growth of microorganisms and degrade the target
pollutants. This can be done by introducing nutrients to stimulate the activity of bacteria in the waste or
by adding new bacteria to the soil. On the basis of removal and transportation of wastes for treatment,
6 www.visionias.in ©Vision IAS
there are two methods of degrading wastes - in-situ bioremediation and ex-situ bioremediation. Hence
option (a) is the correct answer.
• In-situ bioremediation is the application of a biological treatment to clean up hazardous compounds
present in the environment.
• Ex-situ bioremediation describes a process where contaminated soil or water is removed from the
environment by biological organisms. Ex-situ bioremediation can use bioreactors and added nutrients to
speed up the breakdown of environmental pollutants.

Q 18.A
• Statement 1 is correct and Statement 2 is not correct: A national park is an area which is strictly
reserved for the betterment of the wildlife and where human activities like forestry, grazing or
cultivation are not permitted. It is associated with the habitat of wild animal species like Rhino,
Tiger etc. Some examples of national parks are Jim Corbett, Tadoba National Park etc.
• An area, whether within a sanctuary or not, can be notified by the state government to be constituted as a
National Park, by reason of its ecological, faunal, floral, geomorphological, or zoological association or
importance, needed to for the purpose of protecting & propagating or developing wildlife therein or its
environment. No human activity is permitted inside the national park except for the ones permitted by the
Chief Wildlife Warden of the state.
• A wildlife sanctuary is a protected area that is reserved for the conservation only of wildlife - animals
and plant species. Human activities like harvesting or timber, collection of minor forest products and
private ownership rights are allowed. Some examples are Periyar (Kerala), Ranipur (Uttar Pradesh) etc.
• A biosphere reserve is a special area of land or coastal environment in which multiple use of land is
permitted by dividing it into certain zones. The natural or core zones. The natural or core zone consists of
an undisturbed and legally protected ecosystem. The buffer zone surrounds the core area and is managed
to accommodate a greater variety of resource use strategies.
• It takes into consideration the entire ecosystem. The purpose of the formation of the biosphere reserve is
to conserve in situ all forms of life, along with its support system, in its totality, so that it could serve as a
referral system for monitoring and evaluating changes in natural ecosystems.

Q 19.D
• Recently, the Government asked AEPC to take measures to bring about reforms to increase exports
of textile based PPE. The Council request the lifting of ban on export of N95 and PPE suits to
rejuvenate the industry.
• Incorporated in1978, Apparel Export Promotion Council (AEPC) is the official body of apparel
exporters in India that provides assistance to Indian exporters as well as importers/international buyers
who choose India as their preferred sourcing destination for garments. It functions under the Ministry of
Textiles. Hence statement 1 is not correct.
• AEPC is today a powerful body for the promotion and facilitation of garment manufacturing and
their exports. For Indian exporters, AEPC is quite literally a one-stop shop for information
advise technical guidance workforce and market intelligence.
• The Chairman of the Council is elected by the executive committee of the Council. The executive
committee includes persons from the both the Government and the Industry. Hence statement 2 is not
correct.
• AEPC is registered under section 25 of the Companies' Act 1956 and has two categories of Members:
Member Exporter and Registered Exporter.
• Any applicant, firm or society and/or Statutory Organization who is an exporter of garments whether
manufacturer exporter or merchant exporter in the export and aims and objective of the council shall be
eligible to be registered as a Registered Exporter of the Council.

Q 20.C
• Statement 1 is correct: The United Nations Conference on Environment and Development
(UNCED), also known as the Rio de Janeiro Earth Summit, the Rio Summit, the Rio Conference,
and the Earth Summit (Portuguese: ECO92), was a major United Nations conference held in Rio de
Janeiro from 3 to 14 June in 1992.
• Its aim was to put the concept of sustainable development at the forefront of our concerns and to seek
common action to protect the planet from environmental degradation that even threatens to change the
global climate.
• Statement 2 is correct: The three Rio Conventions—on Biodiversity, Climate Change and
Desertification—derive directly from the 1992 Earth Summit. Each instrument represents a way of
7 www.visionias.in ©Vision IAS
contributing to the sustainable development goals of Agenda 21. The three conventions are
intrinsically linked, operating in the same ecosystems and addressing interdependent issues.
• Agenda 21 is a non-binding action plan of the United Nations with regard to sustainable
development. Since 2015, Sustainable Development Goals are included in the newer Agenda 2030.

Q 21.C
• Recent Context: A recent study by TRAFFIC India revealed that Uttarakhand, Maharashtra had highest
leopard poaching in India.
• Trade-Related Analysis of Fauna and Flora in Commerce (TRAFFIC) a wildlife trade monitoring
network, is the leading non-governmental organization working globally on trade in wild animals and
plants in the context of both biodiversity conservation and sustainable development.
• TRAFFIC was established in 1976 by the International Union for Conservation of Nature (IUCN)
and the World Wide Fund for Nature (WWF) to respond to the growing threats posed by illegal
wildlife trade and overexploitation. It is not a bureau under the United Nations Environment Programme
(UNEP). Hence options 1 and 3 are correct.
• India became its member in 1991.

Q 22.D
• Bio-fertilizers are specific types of living organisms like symbiotic bacteria, Cyanobacteria (also called
blue-green algae), seaweeds etc. that can bring about nutrient enrichment of soil in many different ways.
• Anabaena azollae, Anabaena cycadae, Azolla pinnata and Nostoc are different plants that enhance
the productivity of soil when added to it.
• Bacteria like Rhizobium fix nitrogen for plants and Nostoc, Azolla, and Cyanobacteria that are
great nitrogen fixers, are used as biofertilizers in crop fields, most frequently.
• Hence all the options are correct.

Q 23.C
• In bacteria, fungi and lower plants, various kinds of thick-walled spores are formed which help
them to survive unfavourable conditions – these germinate on availability of suitable environment. In
higher plants, seeds and some other vegetative reproductive structures serve as means to tide over
periods of stress besides helping in dispersal – they germinate to form new plants under
favourable moisture and temperature conditions. They do so by reducing their metabolic activity and
going into a state of ‘dormancy’ under favourable conditions.
• In animals, the organism, if unable to migrate, might avoid the stress by escaping in time. The familiar
case of bears going into hibernation during winter is an example of escape in time. Some snails and fish
go into aestivation to avoid summer–related problems-heat and dessication. Under unfavourable
conditions many zooplankton species in lakes and ponds are known to enter diapause, a stage of
suspended development.
• Hence both the options are correct.

Q 24.C
• The CSIR-National Environmental Engineering Research Institute (CSIR-NEERI) is a research institute
created and funded by Government of India.
• It was established in Nagpur in 1958 with focus on water supply, sewage disposal, communicable diseases
and to some extent on industrial pollution and occupational diseases found common in post-independent
India.
• NEERI is a pioneer laboratory in the field of environmental science and engineering and part of
Council of Scientific and Industrial Research (CSIR). Hence statement 1 is correct.
• NEERI has five zonal laboratories at Chennai, Delhi, Hyderabad, Kolkata and Mumbai. NEERI falls
under the Ministry of Science and Technology (India) of central government and is headed by the
Prime Minister as the President and the Minister of Science and Technology as the Vice-
President. Hence statement 2 is not correct and statement 3 is correct.
• NEERI's Vision: Leadership in Environmental Science and Engineering for Sustainable Development.
• NEERI's Mission: CSIR-NEERI would continue to strive for providing innovative and effective
solutions for environmentally sustainable development and to help Government, industry and the society,
especially the 800 million underprivileged people of India.
• The NEERI is an important partner organisation in India's POPs (persistent organic pollutants) national
implementation plan (NIP).

8 www.visionias.in ©Vision IAS


• CSIR-NEERI has successfully transferred two technologies, viz. Phytorid wastewater treatment
technology to 15 MSME entrepreneurs and Solar Electrolytic Defluoridation technology of water to 9
MSME entrepreneurs.
• CSIR-NEERI has developed indigenous “Electronic Nose (e-nose)”, in association with C-DAC
Kolkata, which has a potential to substitute imported “e-nose”. This e-nose consisting of 8 array of
sensors (metal oxide) is useful in monitoring of sulphurous odorants in pulp & paper industry, tannery and
distillery.
• It has also setup emission testing rules for green crackers.

Q 25.B
• Statement 1 is not correct: Thermoregulation is energetically expensive for many organisms. This is
particularly true for small animals like shrews and humming birds. Heat loss or heat gain is a function
of surface area. Since small animals have a larger surface area relative to their volume, they tend to
lose body heat very fast when it is cold outside; then they have to expend much energy to generate
body heat through metabolism. This is the main reason why very small animals are rarely found in
polar regions.
• Statement 2 is correct: Some organisms are able to maintain homeostasis by physiological (sometimes
behavioural also) means which ensures constant body temperature, constant osmotic concentration,
etc. All birds and mammals, and a very few lower vertebrate and invertebrate species are indeed
capable of such regulation (thermoregulation and osmoregulation). Evolutionary biologists believe
that the ‘success’ of mammals is largely due to their ability to maintain constant body temperature
and thrive whether they live in Antarctica or in the Sahara desert.

Q 26.A
• Recent Context: NASA's exoplanet-hunting TESS space telescope has recently completed its primary
mission, but its search for strange new worlds goes on.
• The Transiting Exoplanet Survey Satellite (TESS) is an Explorer-class planet finder recently
launched by NASA. In the first-ever spaceborne all-sky transit survey, TESS will identify planets ranging
from Earth-sized to gas giants, orbiting a wide range of stellar types and orbital distances. The principal
goal of the TESS mission is to detect small planets with bright host stars in the solar neighborhood, so that
detailed characterizations of the planets and their atmospheres can be performed.
• Hence option (a) is the correct answer.

Q 27.C
• Water has been harvested in India since the ancient period. The need of Conservation and Management of
Water was felt even by the people of the Harappa and Mohenjo-Daro, about 5000 years ago. People
harvested rain water and stored it in tanks, they built community lands for collecting water, they harvested
rainwater runoff by capturing water from swollen streams during the monsoon season and by storing it in
water bodies, they harvested water from flooded rivers
• Traditional Water Harvesting Systems in India In respect of availability and non- availability of water,
India can be divided into 15 Ecological regions, ranging from dry, cold desert of Ladakh to the dry hot
desert of Rajasthan, from the subtemperate mountain of the Himalayas to the tropical high mountain of
Nilgiri.
• Table: Traditional Water Harvesting Systems in different Ecological Zones of India
S. No. Ecological Zone Traditional Water Harvesting system/structures
1. Trans- Himalayan Zing
Region (Ladakh)
2. Western Himalayan Region Kul, Naula, Kuhl, Khatri
3. Eastern Himalayan Region Aptani
4. Northern Hill Ranges Zabo, Cheo-oziihi, Bamboo- drip irrigation
5. Brahmputra Valley Dongs, Durgs /Jampois
6. Indo-Gangetic Plain Ahars-pynes, Bengal’s inundation channels, Dighis, Baolis. Etc.
7. Thar Desert Kunds, Kundis, Baoris, Jhalaras, Nadi, Tabas, Tankas, Khadins,
Virdas, Paars etc.
8. Central Highlands Talab, Bandhis, Saza Kuva, Johad, Naada, Bandh,
Tank, Chandella, Bundela Tank etc.
9 www.visionias.in ©Vision IAS
9. Eastern Highlands Katas, Mundas, Bandhas etc.
10. Decan Plateau Cheruvu, Kohli, Tanks, Bhandaras, Phad, Kere, etc.
11. Western Ghats Surangam
12. Western Coastal Planes Virdas
13. Eastern Ghats Korambu
14. Eastern Coastal Plains Eri, Ooranis
15. The Islands Jack Wells
• Hence pairs 1 and 3 are correctly matched.

Q 28.C
• The African Continental Free Trade Area (AfCFTA) is a free trade area, outlined in the African
Continental Free Trade Agreement among African Union nations. AfCFTA is the largest in the world in
terms of participating countries since the formation of the World Trade Organization. Hence statement 1
is correct.
• The agreement was brokered by the African Union (AU) and was signed on by 44 of its 55 member states
in Kigali, Rwanda in 2018. To date, the Agreement has been signed by 54 out of 55 member States.
• Accra, Ghana serves as the Secretariat of AFCFTA. Hence statement 2 is correct.
• The United Nations Economic Commission for Africa (UNECA) forecasts that the simplification of
border controls and the reduction in duties to AfCFTA will boost intra-regional trade by 52 percent in five
years.
• The Indian government and industry has taken several steps to increase investment opportunities and trade
with the African continent. Annual India-Africa Trade Ministers dialogue was launched in 2011 on the
side-lines of the Second India Africa Forum Summit to discuss bilateral trade issues is another important
step. Similarly, India hosted the annual meeting of African Development Bank (AfDB) in 2017. However
the Free Trade Agreement between India and Africa is still being negotiated and not yet
materialised.

Q 29.C
• The Wild Life Protection Act, 1972 is an Act of the Parliament of India enacted for protection of
plants and animal species.
• The Act established schedules of protected plant and animal species; hunting or harvesting these species
was largely outlawed. The Act provides for the protection of wild animals, birds and plants; and for
matters connected there with or ancillary or incidental thereto. It extends to the whole of India. Hence
statement 1 is correct.
• It has six schedules which give varying degrees of protection. Hence statement 2 is correct.
• Schedule I and part II of Schedule II provide absolute protection - offences under these are
prescribed the highest penalties.
• Species listed in Schedule III and Schedule IV are also protected, but the penalties are much lower.
Schedule V includes the animals which may be hunted. The specified endemic plants in Schedule VI are
prohibited from cultivation and planting. The hunting to the Enforcement authorities have the power to
compound offences under this Schedule (i.e. they impose fines on the offenders).

Q 30.A
• Statement 1 is correct: Phosphorus is a major constituent of biological membranes, nucleic acids and
cellular energy transfer systems. Many animals also need large quantities of this element to make shells,
bones and teeth. The natural reservoir of phosphorus is rock, which contains phosphorus in the
form of phosphates.
• When rocks are weathered, minute amounts of these phosphates dissolve in soil solution and are absorbed
by the roots of the plants. Herbivores and other animals obtain this element from plants. The waste
products and the dead organisms are decomposed by phosphate-solubilising bacteria releasing
phosphorus.
• Statement 2 is not correct: Unlike carbon cycle, there is no respiratory release of phosphorus into
the atmosphere. The other two major and important differences between carbon and phosphorus cycle
are firstly, atmospheric inputs of phosphorus through rainfall are much smaller than carbon inputs, and,
secondly, gaseous exchanges of phosphorus between organism and environment are negligible.

10 www.visionias.in ©Vision IAS


Q 31.D
• Recent Context: India joined the World Economic Forum's (WEF) Reskilling Revolution on 22
January which is being held in Davos, Switzerland.
• India joined the Reskilling Revolution Initiative as founding member. The other founding members
include US, UAE, Russia, Pakistan, France and Brazil. Hence statements 1 and 2 are correct.
About Reskilling Revolution Scheme:
• The Reskilling Revolution scheme aims to train workers from technological change and help economies
by providing new skills for the Fourth Industrial Revolution.
• The scheme will act as a public-private platform to generate new employment opportunities.
• The initiative plans to provide one billion people with better education, skills, and jobs by 2030.
Hence statement 3 is correct.
• WEF released a report titled Jobs of Tomorrow: Mapping Opportunity in the New Economy.
• LinkedIn will act as a data partner for the Reskilling Revolution scheme.
• About WEF:
o The World Economic Forum (WEF) was founded in 1971. It is located in Cologny-Geneva,
Switzerland.
o WEF aims to improve the state of the world by engaging business, academic, political, and other
leaders of society to shape regional, global, and industry agendas.
o WEF hosts an annual meeting in the month of January in Davos.
o The founding governments of WEF include India, Brazil, France, Pakistan, the Russian Federation,
the UAE, and the US. The Business partners include Salesforce, PwC, Infosys, The Adecco Group.,
ManpowerGroup, LinkedIn, and Coursera Inc.
Q 32.D
• While addressing the nation from Red Fort on India's 74th Independence Day, the Prime Minister
launched the National Digital Health Mission.
• The NDHM is a holistic, voluntary healthcare programme that will reduce the existing gap between
various stakeholders such as doctors, hospitals and other healthcare providers, pharmacies, insurance
companies, and citizens by bringing them together and connecting them in an integrated digital health
infrastructure. It is a major stride towards achievement of the United Nations Sustainable Development
Goal 3.8 of Universal Health Coverage, including financial risk protection. Hence statement 1 is
correct.
• NDHM has the following components:
o Health ID (HID): The NDHM envisages creating a national health ID for every India. Every
patient who wishes to have their health records available digitally must start by creating a Health
ID. Hence statement 2 is correct.
o Patient Health Record (PHR): A PHR is an electronic record of health-related information on an
individual that conforms to nationally recognized interoperability standards.
o Electronic medical record (EMR) web app: An EMR is best understood as a digital version of a
patient's chart. It contains the patient's medical and treatment history from a SINGLE health facility.
o Digi Doctor Platform (Doctor’s Directory): A single, updated repository of all doctors enrolled in
nation with all the relevant details of the doctors.
o Health Facility Registry: The Health Facility Registry is a single repository of all the health facilities
in the country.
• National Health Authority (NHA), the attached office of the Ministry of Health & Family Welfare has
been given the mandate by the Government of India to design, build, roll-out and implement the NDHM
in the country. Hence statement 3 is correct.
• Hence option (d) is the correct answer.

Q 33.B
Measures to control vehicular pollution include:
• Use of unleaded gasoline: It could reduce the emission of CO and HC and significantly decrease the
quantity of vehicle exhaust particulate matter by 60%. Hence option 1 is not correct.
• Use of low-sulphur petrol and diesel: Low sulphur fuel is essentially cleaner for the environment and
better for engines. It means fewer particulates (fine sooty emissions), which have been linked to asthma
and cancer, and fewer sulphur oxides, which cause acid rain. Hence option 2 is correct.
• Use of catalytic converters in Vehicles: As exhaust emission passes through the catalytic converter,
unburnt hydrocarbons are converted into carbon dioxide and water, while carbon monoxide and nitric
11 www.visionias.in ©Vision IAS
oxide are changed to carbon dioxide and nitrogen gas. Unleaded petrol is the best fuel for automobiles
fitted with catalytic converters to reduce air pollution. Hence option 3 is not correct.
• Use of methanol blended petrol: Methanol is a promising fuel as it is clean, cheaper than fossil fuels and
a good substitute for heavy fuels. A study, in which methanol (M-15) was blended with petrol and used in
the existing BS-IV standard cars, found that the carbon dioxide emission had reduced substantially. Hence
option 4 is correct.

Q 34.B
• Innovation in an organisation is the key to its performance in the field of technology and aids in
production of new age equipment and systems. This is especially true in case of requirements for the
armed forces such as Indian Navy.
• Indian Navy in August, 2020 has launched the Naval Innovation and Indigenisation Organisation
(NIIO) to fulfill its requirements relating to technology. Hence statement 1 is not correct.
• About NIIO:
o The NIIO puts in place dedicated structures for the end-users to interact with academia and
industry. Hence statement 2 is correct.
o These structures strive towards fostering innovation and indigenisation for self-reliance in defence in
keeping with the vision of Atmanirbhar Bharat.
o A three-tiered organisation:
▪ Naval Technology Acceleration Council (N-TAC): It will bring together the twin aspects of
innovation and indigenisation and provide apex level directives.
▪ A working group under the N-TAC will implement the projects.
▪ A Technology Development Acceleration Cell (TDAC): It has been created for induction of
emerging disruptive technology in an accelerated time frame
Q 35.A
• There are many non-governmental organizations that oppose cruelty to animals. People for the
Ethical Treatment of animals (PETA) & People for Animals (PFA) are two such organizations.
• These organizations work for the welfare of animals. Animal lovers across the world are of the opinion
that animals too have rights to live and develop peacefully like human beings.
• In India Legal protection is offered to animals through the Prevention of Cruelty to Animals (PCA)
Act- 1960 which was enacted in December 1960 with the objective of preventing infliction of
unnecessary pain and suffering to animals.
• In exercise of powers conferred by this Act, the central Government made the Prevention of Cruelty to
Animals (Establishment and Regulation of Societies for Prevention of Cruelty to Animals) Rules,
2001.
• Accordingly, Societies for Prevention of Cruelty to Animals (SPCAs) have been organized on state
and district level across the country. The Animal Welfare Board of India, at Chennai, conducts and
co-ordinates various activities pertaining to rescue, rehabilitation, care and welfare of animals. We
should take care of animals and encourage others to do the same.
• The Prevention of Cruelty to Animals (PCA) Act- 1960 contains following Rules-
o Performing Animals Rules – 1973 and Performing Animals (registration) Rules-1972.
o Transport of Animals Rules-1978 and 2001.
o Prevention of Cruelty (slaughter house) Rules-2000.
o Prevention of Cruelty to Animals (establishment and regulation of society for prevention of cruelty to
animals) Rules-2001.
o Breeding of and Experiments on Animals (control and supervision) Rules-1998 as amended in
February 2001. Hence the correct option is (a)
Q 36.B
• Interspecific interactions arise from the interaction of populations of two different species. They
could be beneficial, detrimental or neutral (neither harm nor benefit) to one of the species or both.
Assigning a ‘+’ sign for beneficial interaction, ‘-’ sign for detrimental and 0 for neutral interaction, let us
look at all the possible outcomes of interspecific interactions.
• Both the species benefit in mutualism and both lose in competition in their interactions with each other. In
both parasitism and predation, only one species benefits (parasite and predator, respectively) and the
interaction is detrimental to the other species (host and prey, respectively). The interaction where one
species is benefitted and the other is neither benefitted nor harmed is called commensalism. In
amensalism, on the other hand, one species is harmed whereas the other is unaffected. Predation,
parasitism and commensalism share a common characteristic– the interacting species live closely
together.
12 www.visionias.in ©Vision IAS
Q 37.A
• For aquatic organisms the quality (chemical composition, pH) of water becomes important. The
salt concentration (measured as salinity in parts per thousand), is less than 5 in inland waters, 30-35 in the
sea and > 100 in some hypersaline lagoons.
• Some organisms are tolerant of a wide range of salinities (euryhaline) but others are restricted to a narrow
range (stenohaline). Many freshwater animals cannot live for long in seawater and vice versa because of
the osmotic problems, they would face.
• Euryhaline organisms are able to adapt to a wide range of salinities. An example of a euryhaline fish
is the molly which can live in freshwater, brackish water, or saltwater. The green crab is an example of a
euryhaline invertebrate that can live in salt and brackish water. Hence option (a) is the correct answer.
• Stenohaline describes an organism, usually fish, that cannot tolerate a wide fluctuation in the salinity of
the water.

Q 38.A
• Habitat deterioration and destruction is the major cause of threat to many wild plant and animal species
and plant communities. Habitat destruction could be caused as a result of:
o Clearing of forest areas for settlement or agricultural expansion, commercial lodgings, large hydel
schemes, fire, human and livestock pressure, etc.
o Pollution (both air and water) stresses ecosystem, mismanagement of industrial and agriculture
wastes threaten both terrestrial and aquatic ecosystem.
o Over exploitation, mainly for commercial purposes like meat, fur, hides, body organs, medicine, etc.
o Accidental or deliberate introduction of exotic species which can threaten native flora and fauna
directly by predation or by competition and also indirectly by altering the natural habitat or
introducing diseases.
o Fragmentation of forests into subdivisions destroys habitat for wildlife, including rare species such
as the Florida black bear and the red-cockade woodpecker. Some wild animals like bears, bobcats and
foxes are unable to adapt to changes in the forest.
o Critical natural habitat and its associated biodiversity are diminished in wars around the world.
Similarly, actions such as deforestation, habitat destruction and degraded human waste disposal-all
associated with war and its aftermath can affect other key ecosystem services such as erosion control,
water quality and food production.
o Destruction of habitats is caused on account of land being taken by new roads. This prevents free
movement of animals or birds from their habitats either in search of food or for the purpose of
migration from one place to another.
o Wounding and killing of animals hit by motor vehicles and trains, or victims from drowning when
falling into steep-sided canals.
o Fish are also affected. Salmon, for instance, are frequently indirectly affected by roads. Impervious
edges to roads, such as concrete, increase the flow of water from the road into streams, leading to a
build-up of sediment, increased water temperature and pollution. Water running off from roads is the
biggest hazard, as salmon are very sensitive to these irregular "flash flows".
o Disturbance to animals arising from noise, light, movement and human activity. For example, to
breed successfully, songbirds require a noiseless atmosphere. Hence correct answer is option (a).

13 www.visionias.in ©Vision IAS


Q 39.A
Particulate matter/ particulate pollutants:
• They are both liquid and solid aerosols that remain suspended in the atmosphere. Hence statement 1 is
correct.
• The effect is of particulate pollutants are largely dependent on the particle size. Air-borne particles such as
dust, fumes, mist etc., are dangerous for human health. Hence statement 2 is not correct.
• Particulate pollutants bigger than 5 microns are likely to lodge in the nasal passage, whereas particles of
about 10 microns enter into lungs easily.
• These are present in vehicle emissions, smoke particles from fires, dust particles and ash from industries.
• Particulates in the atmosphere may be viable or non-viable.
• Viable pollutants are the organic substances or minute living organisms which are spreaded in the
atmosphere and cause harmful effects for ex: bacteria, fungus, viruses and other organic particulates,
while the non viable pollutants are the inorganic materials or non living particles like, fog, mist, dust ,
fumes etc.
Q 40.A
• The United Nations defines the desertification as- Land degradation in arid, semi-arid and dry sub-
humid areas brought about by factors such as climatic variations and human activities, is called
desertification.
Causes of Desertification:
• Massive deforestation for various activities like an extension of agriculture, pasture development,
industrialization, mining, urbanization etc.
• Overgrazing by cattle that renders the earth- surface bare, infertile, and thus unproductive.
• Over cultivation that depletes most of the organic content of the soil and makes it prone to soil erosion
by wind and water.
• Growth of human population and consequent degradation of land through increasing human activities
• Unscientific irrigation practices and flooding of fields makes the soil saline and unproductive.
• Crop rotation can improve yield and profitability over time, control weeds, break disease cycles, limit
insect and other pest infestations, provide an alternative source of nitrogen, reduce soil erosion, increase
soil organic matter, improve soil tilth, and reduce runoff of nutrients and chemicals.
• Organic manures improve the soil texture, allowing it to hold water longer, and increase the bacterial and
fungal activity in the soil. It contributes to combating desertification by preventing soil erosion.
• Hence correct answer is option (a).

Q 41.D
• Tillage is the agricultural preparation of soil by mechanical agitation of various types, such as digging,
stirring, and overturning.
• Farmers get following benefits from tilling the lands.
o Tillage removes the weeds that grow in crop fields.
o It makes the soil loose by breaking up the surface layers.
o It provides a bed of soil which holds sufficient moisture continuously over many years.
o Proper tillage facilitates the aeration of soil. Aeration provides free circulation of oxygenand
water, results in the increased biological activities in the soil and allows the Nitrogen Fixing
Bacteria to do their duties.
o It contributes to the health of plants by inhibiting plant diseases and by discouraging the developments
of various types of insects that harm plants.
o Proper tillage practices help the conservation of soil in many ways. The uphill and downhill direction
of furrows promotes soil erosion. But, by ploughing across the slope, water is directed to enter into
the soil. This type of ploughing is known as contour ploughing.
• Hence correct answer is option (d).

Q 42.D
• Pyrolysis is the thermal decomposition of materials at elevated temperatures in an inert condition in
an oxygen-deficient environment.
• It involves a change in chemical composition. The word is coined from the Greek-derived elements pyro
“fire” and lysis “separating”.
• It is most commonly used in the treatment of organic materials. It is one of the processes involved in
charring wood.
14 www.visionias.in ©Vision IAS
• It is considered as the first step in the processes of gasification or combustion.
• The process is used heavily in the chemical industry, for example, to produce ethylene, many forms of
carbon, and other chemicals from petroleum, coal, and even wood, to produce coke from coal.
• Aspirational applications of pyrolysis would convert biomass into syngas and biochar, waste plastics
back into usable oil, or waste into safely disposable substances.
• Hence option (d) is the correct answer.
• Additional information:
o Fermentation: It is the chemical breakdown of a substance by bacteria, yeasts, or other
microorganisms, typically involving effervescence and the giving off of the heat.
o Bio-methanation: It is a process by which organic material is microbiologically converted under
anaerobic conditions to biogas.
o Incineration: It is a way to treat waste through controlled burning. Waste is shredded and heated to
over 1025 degree Celsius in a furnace.
Q 43.D
• Healthy ecosystems are the base for a wide range of economic, environmental and aesthetic goods and
services. The products of ecosystem processes are named as ecosystem services, for example, healthy
forest ecosystems purify air and water, mitigate droughts and floods, cycle nutrients, generate fertile
soils, provide wildlife habitat, maintain biodiversity, regulates climate, pollinate crops, provide storage
site for carbon and also provide aesthetic, cultural and spiritual values.

Q 44.D
• The National Green Corps (NGC), popularly known as a programme of Eco clubs is a nationwide
initiative of the Ministry of Environment, Forests and Climate Change Hence statement 2 is not
correct
Objectives:
• To impart knowledge to school children, through hands-on experience, about their immediate
environment, interactions within it and the problems therein. Hence statement 1 is not correct.
• To develop requisite skills of observation, experimentation, survey, recording, analysis and reasoning for
conserving the environment through various activities.
• To inculcate the proper attitude towards the environment and its conservation through community
interactions.
• To sensitize children to issues related to environment and development through field visits and
demonstrations.
• To promote logical and independent thinking among children so that they are able to make the right
choices in a spirit of scientific inquiry.
• Structure:
o There is District Implementation and Monitoring Committee to supervise, organise training for In-
charge teachers, and monitor periodically the implementation of scheme at the District level.
o There is a State Steering Committee for guidance, direction and to oversee the implementation of
the scheme.
o The State Nodal Agency coordinates the implementation of the scheme in the State and organize
related activities like training to Master Trainers.
o The National Steering Committee will give overall direction to the programme and ensure linkages
at all levels.
Q 45.A
• The accelerated rates of species extinctions that the world is facing now are largely due to human
activities. There are four major causes (‘ The Evil Quartet ’ is the sobriquet used to describe them).
o Habitat loss and fragmentation: This is the most important cause driving animals and plants to
extinction. This habitat loss mainly happens due to tropical rain forests. For eg. The Amazon rain
forest (it is so huge that it is called the ‘lungs of the planet’) harbouring probably millions of species is
being cut and cleared for cultivating soya beans or for conversion to grasslands for raising beef cattle.
Besides total loss, the degradation of many habitats by pollution also threatens the survival of many
species. When large habitats are broken up into small fragments due to various human activities,
mammals and birds requiring large territories and certain animals with migratory habits are badly
affected, leading to population declines.
o Over-exploitation: Humans have always depended on nature for food and shelter, but when ‘need’
turns to ‘greed’, it leads to over-exploitation of natural resources. Many species extinctions in the last
500 years (Steller’s sea cow, passenger pigeon) were due to overexploitation by humans.
15 www.visionias.in ©Vision IAS
o Alien species invasions: When alien species are introduced unintentionally or deliberately for
whatever purpose, some of them turn invasive, and cause decline or extinction of indigenous species.
The Nile perch introduced into Lake Victoria in east Africa led eventually to the extinction of an
ecologically unique assemblage of more than 200 species of cichlid fish in the lake; introduction of
the African catfish Clarias gariepinus for aquaculture purposes is posing a threat to the indigenous
catfishes in our rivers.
o Co-extinctions: When a species becomes extinct, the plant and animal species associated with it in an
obligatory way also become extinct. When a host fish species becomes extinct, its unique assemblage
of parasites also meets the same fate. Another example is the case of a coevolved plant-pollinator
mutualism where extinction of one invariably leads to the extinction of the other.
• Hence, all are correct options.

Q 46.D
• Radiation Pollution or Radioactive pollution can be defined as the release of radioactive substances or
high-energy particles into our natural resources as a result of human activity, either by accident or by
design. It is the uncontrolled distribution of radioactive material in a given environment.
• Radioactive substances emit rays like alpha (α) rays, gamma (γ) rays and beta (β)rays. These rays are
emitted from unstable radioactive substances so as to become stable. In this way, these are gradually
reduced in their value.
• The time taken by a radioactive substance to get reduced up to the half of its initial value is called a
half-life period.
• The unit applied to express the decay of a radioactive substance is called as Becquerel.
• Radiation pollution may be caused by both –
o Natural radiations
o Man-made radiations.
• Cosmic radiations, ultraviolet radiations etc. are examples of Natural Radiations.
• The application of explosive devices, testing of bombs, leakage from nuclear reactors, mining of
radioactive material, use of nuclear weapons etc. are man-made sources of radiation pollution
• Hence option (d) is the correct answer.

Q 47.C
• Recent Context: Merchandise Export from India Scheme (MEIS), a key incentive scheme for exports,
will be wound up by December 31, 2020 as the government has found it to have failed to deliver and not
yield the desired result of boosting exports which have hovered around $300 billion in the last five years
despite its
liberal application across sectors. Government sources said the liability under Merchandise Export from
India Scheme ballooned from Rs 20,000 crore to about Rs 45,000 crore in FY20 “reaching an
unsustainable level”.
• Merchandise Exports from India Scheme (MEIS) under Foreign Trade Policy of India (FTP 2015-
20) is one of the two schemes introduced in Foreign Trade Policy of India 2015-20, as a part of
Exports from India Scheme. The other scheme is SEIS, Service Exports from India Scheme. Hence
statement 1 is correct.
• The Government of India brought in the Merchandise Exports Incentive Scheme (MEIS), replacing five
other similar incentive schemes present in the earlier Foreign Trade Policy 2009-14. The schemes that
have been replaced by the MEIS scheme include:
o Focus Product Scheme (FPS)
o Focus Market Scheme (FMS)
o Market Linked Focus Product Scheme (MLFPS)
o Agri. Infrastructure incentive scheme
o Vishesh Krishi Gramin Upaj Yojna (VKGUY)
• As per the present FTP, the MEIS scheme does not aim to merely replace these five schemes but also aims
to rationalize the incentives and enlarges their scopes by removing various restrictions.
• The objective of the MEIS Scheme is to offset infrastructural inefficiencies and the associated costs of
exporting products produced in India giving special emphasis on those which are of India’s export interest
and have the capability to generate employment and enhance India’s competitiveness in the world market.
• About the Scheme: With the aim in making India’s products more competitive in the global markets, the
scheme provides incentive in the form of duty credit scrip to the exporter to compensate for his loss
on payment of duties. Hence statement 2 is correct.

16 www.visionias.in ©Vision IAS


• The incentive is paid as percentage of the realized FOB value (in free foreign exchange) for notified
goods going to notified markets. To determine the quantity of incentive, the countries have been
segregated into three groups. Incentives on export of each product at 8-digit level (ITC HS codes), depend
on the group in which its destination country belong.
• There are essentially three country groups. Group A has India’s traditional destinations such as the EU
countries and USA. Group B has the maximum number of countries and covers almost all of India’s major
export destinations globally. It is worth mentioning here that Group B has the highest quantum of
incentive. Group C, on the other hand, has no incentive at all. It can be divided into, SAARC, Australia
and New Zealand, some EU and African countries.

Q 48.C
• Recent Context: An amendment has been done in the International Solar Alliance Agreement to
give membership to all UN member countries. Earlier the membership was restricted to solar-
resource-rich countries (which lie either completely or partly between the Tropic of Cancer and the
Tropic of Capricorn) to address their special energy needs.
• The International Solar Alliance was launched jointly by the Hon’ble Prime Minister of India and the
President of France during COP21 in Paris in 2015. It aims to contribute to the implementation of the
Paris Climate Agreement through rapid and massive deployment of solar energy. As on 30 July 2020, 87
Countries had signed the Framework Agreement of the ISA and of these 67 have deposited their
instruments of ratification.
• The ISA aims to bring together countries to provide a collective response to the main common obstacles
to the massive deployment of solar energy in terms of technology, finance and capacity. The objective of
the ISA is to mobilize member countries, seek commitments from international organisations and
mobilize private sector, to support rural and decentralised applications, access to affordable finance, island
and village solar mini grids, rooftop installations, and solar e-mobility technologies.
• The ISA is the first international body to have a secretariat in India. Although it was founded in
Paris, France, its headquarters is in Gurugram, India.
• Hence both the statements are correct.

Q 49.C
• Water (Prevention & Control of Pollution) Act, 1974 is comprehensive legislation that regulates
agencies responsible for checking on water pollution and ambit of pollution control boards both at
the centre and states. The Water (Prevention & Control of Pollution) Act, 1974 was adopted by the
Indian parliament with the aim of prevention and control of Water Pollution in India. Some of the
important sections regulating the prevention of water pollution as per the act are as discussed below.
• Functions of the State Board:
o Section 17 of the Water (Prevention & Control of Pollution) Act, 1974 clearly lists all functions of the
respective state boards for countering water pollution. The state board of respective states is
empowered to plan a comprehensive program for the prevention, control or abatement of pollution of
streams and wells, collect and disseminate information relating to water pollution and encourage,
conduct and participate in investigations and research relating to problems of water pollution and
prevention. i.e to review the matter relating to the identification and notification of disposal sites.
Hence statement 1 is correct.
o Section 25 of the Water (Prevention & Control of Pollution) Act, 1974 states that Prior Consent of the
State Board under section 25 is necessary to set up any industry, plant or process which is likely to
discharge sewage or trade effluent into a stream or well or sewer or on land or bring into use any new
or altered outlets for the discharge of sewage or begin to make any new discharge of sewage.
o Power to take emergency measures section 32 of the Water (Prevention & Control of Pollution) Act,
1974 describes the power to take emergency measures in case of pollution of stream or well. Under
the act, State Board may issue orders to remove the matter, which is, or may cause pollution; or
remedy or mitigate the pollution, or issue prohibition orders to the concerned persons from
discharging any poisonous or noxious or polluting matter. Hence statement 3 is correct.
o It also has powers to cancel the authorization or suspend it, if it is found that the occupier of
disposal site is not following the Provisions of Rules. Hence statement 2 is correct.
o Section 24 and 43 of the Water (Prevention & Control of Pollution) Act, 1974 relate to prohibition on
use of stream or well for disposal of polluting matter and penalty for contravention thereof.

17 www.visionias.in ©Vision IAS


Q 50.B
• Canada's 4,000-year-old Milne Ice Shelf, the last fully intact ice-shelf has broken apart due to
melting from both hotter air above and warmer water below. Much of the Earth's ice shelves have
collapsed because of rising temperatures in both poles.
• The Milne Ice Shelf is at the fringe of Ellesmere Island, in the sparsely populated northern
Canadian territory of Nunavut.
• Hence option (b) is the correct answer.

Q 51.A
• Biological Oxygen Demand is defined as the amount of dissolved oxygen required by aerobic
microorganisms to breakdown the organic materials in a sample of water at a specific temperature
& timeframe. Hence statement 1 is correct.
• The amount of BOD in the water is a measure of the amount of organic material in the water, in terms of
how much oxygen will be required to break it down biologically.
• Biological oxygen demand can be used as an indicator of the level of environmental pollution.
• Clean Water would have a BOD value of less than 5 ppm whereas highly polluted water could have
a BOD value of 17 ppm or more. Hence statement 2 is not correct.

Q 52.D
• The soil is essential for the growth and development of vegetation that controls most of the environmental
processes. Secondly, it is the basic resource for agricultural production. The process of soil formation may
take thousands of years but the same soil is taken away within a very short period of time by natural
forces like wind and rain.
• Human activities of clearing of vegetation, overgrazing by cattle, careless mining activities etc. facilitate
the process of soil erosion which severely alters the functions of soil and spoils the landscape heavily.
• The Soil Erosion alters Soil Functions in Following Ways:
o It destroys the ability of soil to act as a filter for pollutants and hence there is an increase in the
quantity of pollutants. Hence statement 1 is correct.
o It destroys the ability of soil to keep up the bio-geo-chemical cycles like the water cycle and the
nitrogen cycle. Hence statement 2 is correct.
o It destroys the ability of soil to provide habitat and support the biodiversity at a
particular place. Hence statement 3 is correct.
• So far, about 2000 million ha of soil has been degraded through human activities. Soil erosion by water
has alone degraded about 56 per cent of soil. Degradation of soil through wind erosion, overgrazing,
clearing of vegetation and industrial activities are 28, 35, 7 and 1 per cent respectively. In light of these
facts, we can say that soil erosion and the spoilage of landscape are inter-related.
• The erosion of soil can be controlled in the following ways-
o By mechanical protection of soil like the building of bunds and terraces along hill slopes and other
slopey areas to control the speed of surface runoff.
o By biological methods of soil protection like planting of trees, checking of deforestation and
controlling overgrazing by cattle.
o By water conservation and integrated system of soil protection, like protecting the land from
waterlogging and adopting proper methods of land tilling.
Q 53.C
• Due to the rapid decline in the wildlife population, the Government of India during 1952 had
constituted an advisory body designated as the Indian Board for Wildlife (IBWL). The Indian Board
for Wildlife was chaired by the Prime Minister.
• During the 1970s the Government of India appointed a committee for recommending legislative measures
and administrative machinery for ensuring environmental protection. Accordingly, comprehensive central
legislation was enacted in 1972 called the Wildlife (Protection) Act for providing special legal protection
to our wildlife and to the endangered species of fauna in particular.
• As per the amendment to the Wildlife (Protection) Act, 1972 in 2002, a provision was incorporated
for the constitution of the National Board for Wildlife, replacing the Indian Board for
Wildlife. Hence statement 1 is not correct.
• National Board for Wildlife (NBWL) is a statutory Board constituted on 22nd September 2003 under
Section 5 of the Wild Life (Protection) Act, 1972. The NBWL is chaired by the Hon’ble Prime
Minister. Hence statement 2 is not correct.
• The board advises the federal and state governments in matters concerning wildlife conservation policy,
illegal trade and poaching, management of national parks and sanctuaries, impact assessments of projects
18 www.visionias.in ©Vision IAS
on wildlife, and other related issues. The tenure of each board is 3 years, after which a new one is
constituted.
• It examines proposals for non-forestry use of forestland within National Parks and Wildlife Sanctuaries,
and issue or deny permissions. These include, among others, proposals for highways, dams, industries and
mining and no alternation of boundaries in national parks and wildlife sanctuaries can be done
without the approval of the NBWL. Hence statement 3 is correct.
Q 54.B
• A World Industry Conference was organized in 1984 by the United Nations in Canada, after a
series of industrial accidents in different parts of the world. Hence option (b) is the correct option.
• It was the first attempt to provide a code of conduct for sound management in the business sector. As a
result of this, the concept of Sustainable Development and Eco- friendly Technology or Eco-efficiency
was introduced in the area of development.
• Some of the major objectives of eco-efficiency are
o Judicious use of fossil fuels,
o Reduction of pollution load,
o Maximum production
o Reduction of man-power-wastage; and
o Sustainable Development.
Q 55.D
• Ozone-depleting substances are chemicals that destroy the earth’s protective ozone layer. They include:
• Chlorofluorocarbons (CFCs)
• Halons
• Carbon tetrachloride (CCl4)
• Methyl chloroform (CH3CCl3)
• Hydrobromofluorocarbons (HBFCs)
• Hydrochlorofluorocarbons (HCFCs)
• Methyl bromide (CH3Br)
• Bromochloromethane (CH2BrCl)
• Production and import of these chemicals is controlled by the Montreal Protocol on Substances that
Deplete the Ozone Layer (the Montreal Protocol).
• Hydrofluorocarbons (HFCs) are greenhouse gases (GHGs) commonly used in a wide variety of
applications, including refrigeration, air-conditioning (AC), building insulation, fire extinguishing
systems, and aerosols. HFCs have high global warming potential (GWP), raising concern about their
impacts as they become increasingly used as replacements for ozone-depleting substances
• Hydrofluorocarbons (HFCs), was introduced as non-ozone depleting alternatives to support the
timely phase-out of CFCs and HCFCs.
• Hence option (d) is the correct answer. These chemicals do not deplete the stratospheric ozone layer,
some of them have high Global Warming Potential, ranging from 12 to 14,000. Therefore HFCs are
potent greenhouse gases that can be hundreds to thousands of times more potent than carbon dioxide
(CO2) in contributing to climate change per unit of mass.

Q 56.B
• Pradhan Mantri Awas Yojana – Gramin (PMAY-G), earlier known as Indira Awaas Yojana (IAY),
is a scheme for rural housing by the Indian Government. This social welfare program was initiated to meet
the objectives of the “Housing for All” scheme by the year 2022.
• The main features of the scheme of PMAY-G are:
o Identification of beneficiaries as per the housing deprivation parameters and exclusion criteria
prescribed under Socio-Economic Caste Census (SECC) 2011 after due verification by Gram
Sabha. Hence statement 2 is correct.
o Providing assistance for construction of 2.95 crore pucca houses for eligible rural households by
March 2022 in phases.
o Enhancement of unit assistance from Rs. 70,000 (IAY) to Rs. 1.20 lakh in plain areas and from Rs.
75,000(IAY) to Rs.1.30 lakh in Hilly States, NE States, difficult areas and IAP districts.
o It is a Centrally Sponsored Scheme. The cost of unit assistance is to be shared between Central and
State Governments in the ratio 60:40 in plain areas and 90:10 for North Eastern and hilly states.
Hence statement 1 is not correct.
o Provision of assistance for construction of toilets amounting to Rs. 12,000/- through Swach Bharat
Mission-Gramin, MGNREGS or any other dedicated source of funding and 90/95 days of unskilled
wages under MGNREGS over and above the unit assistance.
19 www.visionias.in ©Vision IAS
o Enhancement in the minimum unit size of the house from 20 sq.m.(IAY) to 25 Sq.m.
o Facilitating willing beneficiaries to avail loan from Financial Institutions for an amount of up to Rs
70,000
• Recently the programme is suffering a financial setback as state governments have refused to release
their share of funds. It is because the state governments have lost tax revenues because of economic
slowdown due to Covid-19 pandemic.

Q 57.D
• Methane is produced by the breakdown or decay of organic material and can be introduced into the
atmosphere by either natural processes – such as the decay of plant material in wetlands, the seepage
of gas from underground deposits or the digestion of food by cattle – or human activities – such as
oil and gas production,coal mines , rice farming or waste management.
• Fossil fuel production, distribution and use:
The largest human source is from the production, distribution and combustion of fossil fuels. This creates
33% of human methane emissions.
• Livestock farming
An important source of methane emissions is from enteric fermentation in farm animals. This creates 27%
of human methane emissions.1 Animals like cows, sheep and goats are examples of ruminant animals.
During their normal digestion process they create large amounts of methane. Enteric fermentation occurs
because of microorganisms in the stomach of these animals. This creates methane as a by-product that is
either exhaled by the animal or released via flatus.
• Landfills and waste
Another important human source of methane emissions is from landfills and waste. Methane gets
generated by the decomposition of solid waste in landfills. This also happens with animal and human
waste streams. This accounts for 16% of human methane emissions.
• Paddy fields : Paddy fields for rice production are man-made wetlands. They have high moisture content,
are oxygen depletion and have ample organic material. This creates a great environment for methane
producing microbes that decompose the organic matter.
• Hence option (d) is correct.

Q 58.B
• Conservation does not mean preservation, but it means utilisation of natural resources in such a way that
they are not destroyed completely and can be further utilised by future generations.
• Modern conservation measures have three main objectives:
o To preserve biological diversity;
o To maintain essential ecological processes and life support systems; and
o To ensure sustainable utilisation of species and ecosystems.
• The following two strategies are used for conservation of biodiversity and wildlife:(a) In-situ
conservation, (b) Ex-situ conservation
• The in-situ conservation means conservation of wildlife in its natural habitat. It emphasises the protection
of the ecosystems of the original habitats or natural environment.
• The in-situ approach includes the protection of a typical ecosystem through a network of protected
areas on land or sea. Hence statement 1 is not correct.
• These are managed through state or other effective agencies. The biosphere reserves, national
parks and wildlife sanctuaries are included in the protected areas. Hence statement 2 is correct.
• The ex-situ conservation means conservation of wildlife outside its natural habitat. The conservation takes
place in captivity under the supervision of experts. Generally, botanical gardens, zoos, aquariums,
parks, agricultural research centres, forest research centres, etc. are the artificial habitats for ex-
situ conservation.
Q 59.D
• Pneumoconiosis: It is one of a group of interstitial lung disease caused by breathing in certain kinds of
dust particles like coal dust, silica and asbestos etc. that damage the lungs. Hence pair 1 is correctly
matched.
• Byssinosis: It is a disease of the lungs. It is caused by breathing in cotton dust or dust from other
vegetable fibres such as flax, hemp, or sisal while at work. Hence pair 2 is correctly matched.
• Minamata disease: It is a neurological syndrome caused by severe mercury poisoning. Hence pair 3 is
correctly matched.

20 www.visionias.in ©Vision IAS


Q 60.D
• Environmental Impact Assessment has been defined as an activity that has been designed to identify,
predict and interpret the impact of an action on human health, including the well being of ecosystems on
which the survival of human beings depends.
• EIA was introduced in India in 1978. EIA covers the following projects:
• Mandatory EIA: Major projects that are mandatorily subjected to EIA before execution include crude
oil refineries, thermal power stations, chemical installations, trading ports, special waste incineration,
installations for the disposal of radioactive wastes, motorways and major roads, waste treatment or
landfill, iron and steel industry, heavy mining, etc.
• Discretionary EIA: Some projects are subjected to discretionary environmental assessment in case they
are likely to have significant environmental effects. Such projects include poultry units, metal processing,
mineral extracting, glass making, food manufacturing, holiday resorts, tanneries, paper industry etc.
• Hence option (d) is the correct answer.

Q 61.D
• Statements 1 is not correct: In most ecosystems, all the pyramids, of the number, of energy and biomass
are upright, i.e., producers are more in number and biomass than the herbivores, and herbivores are more
in number and biomass than the carnivores. The pyramid of biomass in the sea is generally inverted
because the biomass of fishes far exceeds that of phytoplankton.
• Statement 2 is not correct: An inverted pyramid of numbers can be found in an ecosystem where the
community contains a few producers with very large biomass that support a larger number of smaller
consumers. Also, energy at a lower trophic level is always more than at a higher level.
• Statement 3 is not correct: Pyramid of energy is always upright, can never be inverted,
because when energy flows from a particular trophic level to the next trophic level, some energy is always
lost as heat at each step. Each bar in the energy pyramid indicates the amount of energy present at each
trophic level in a given time or annually per unit area.
Q 62.D
• Ridge or elevated part of a relatively shallow area of the seafloor approaching the surface of the sea
is called as a coral reef.
• Corals are massive wave resistant structures built largely by coral and consisting of skeletal and
chemically precipitated material. These are made by polyps i.e. animals belonging to the
class Coelenterata.
• There are basically three types of coral reefs- Fringing reefs, Barrier reefs and Atoll.
o The fringing reefs develop along the shoreline of an island and usually extend outwards.
o The barrier reefs develop away from the shoreline and form a channel or lagoon between the reef
and the shore.
o The atolls are islands made of corals.
• In Indian seas, coral reefs are found in the Gulf of Mannar, Pak-bay, Lakshadweep (atolls), Gulf of
Kutch, and Andaman and Nicobar sea areas. The health of coral reefs has been declining over the past
several decades. The pollution of seawater, destructive fishing practices, algal blooms, and eutrophication,
coral diseases and coral bleaching are some serious causes of depletion of coral reefs.
• What is Coral Bleaching? The whitening of coral colonies due to the loss of symbiotic zooxanthellae
from the tissues of polyps is called Coral Bleaching.
• Zooxanthellae are unicellular algae that provide colour to corals. They also provide food to corals and
their deaths lead to the breakdown of the symbiotic relationship between them. This breakdown of the
symbiotic relationship causes starvation and resultant deaths of coral polyps. This condition exposes the
white calcium carbonate skeletons of the coral colony.
• There are a number of stresses or environmental changes that may cause bleaching. These causes include
disease, excess shade, increased levels of ultraviolet radiation, sedimentation, pollution, salinity changes,
and increased temperatures.
• Other causes of coral bleaching include -
o Increased exposure to ultraviolet (UV) radiation;
o Large amounts of stormwater from heavy rains flooding the reef;
o The exposure of coral to certain chemicals or diseases;
o Sediments such as sand or dirt covering the coral;
o Excess nutrients such as ammonia and nitrate from fertilizers and household products entering the
reef ecosystem. (The nutrients might increase the number of zooxanthellae in the coral, but it is
possible that the nutrient overload increases the susceptibility of coral to diseases.) Hence option (d)
is the correct answer.
21 www.visionias.in ©Vision IAS
Q 63.B
• While the addition of nitrogen to the soil may be necessary, farmers often use more than what's
recommended, and over time this excess nitrogen pollutes water and air. Nitrogen at higher levels causes a
loss of certain plant species, depletion of soil nutrients, death of fish and aquatic organisms, and
contamination of drinking water.
• Impacts of excessive use of nitrogen-based fertilizers:
o Soil Imbalance: In the soil, too much nitrogen also creates an imbalance of nutrients that causes a
depletion of other important minerals such as calcium, phosphorus and magnesium. This also kills
the essential soil organisms. Hence option 1 is correct.
o Algae Growth/Eutrophication: When nitrogen levels in rivers and streams increase, they aid in
algae overgrowth. As algae die and decompose, organic matter in the water increases. This process
uses up oxygen, causing levels to drop. Without the oxygen, fish, crabs and other aquatic life die.
o Plant Loss: Though nitrogen serves to aid plants in their growth, weeds and non-native plants tend
to grow more readily with additional nitrogen supplies. Other plants that have lower nitrogen
needs end up dying, causing a decline in native species, according to the Ecological Society of
America. In California, for example, the National Science and Technology Council reports that too
much nitrogen encourages the growth of non-native grasses and kills off lichens on trees. In the
coastal areas of the western United States, soils have higher levels of nitrogen, which feed non-native
grasses. Hence option 2 is correct.
o Water Contamination: A soluble substance, nitrogen soaks deeply into the soil after a rainstorm or
after irrigation, reaching groundwater and nearby wells. When babies under a year old and elderly
people ingest water with high nitrogen levels, they can develop symptoms such as gastrointestinal
swelling and irritation, diarrhoea, and protein digestion problems. These symptoms result from the
condition methemoglobinemia, also called "blue baby syndrome," which occurs when nitrates mix
with iron in red blood cells, leaving the blood unable to transport oxygen to the body's cells. Hence
option 3 is correct.

Q 64.B
• Statement 1 is not correct: A mycorrhiza is a symbiotic association between a green plant and a
fungus. The plant makes organic molecules such as sugars by photosynthesis and supplies them to the
fungus, and the fungus supplies to the plant water and mineral nutrients, such as phosphorus, taken from
the soil. It is the lichen that is a composite organism that arises from algae or cyanobacteria living among
filaments of multiple fungi species in a mutualistic relationship.
• Statement 2 is correct: Mycorrhizae are particularly important in assisting the host plant with the
uptake of phosphorus and nitrogen, two nutrients vital to plant growth. Mycorrhizae actually
increase the surface area associated with the plant root, which allows the plant to reach nutrients and water
that might not be available otherwise.

Q 65.B
• When a forest is cut and burned to establish cropland and pastures, the carbon that is stored in the
tree trunks joins with oxygen and is released into the atmosphere as CO2.
• Deforestation affects the local climate of an area by reducing the evaporative cooling that takes place from
both soil and plant life. Evaporation and evapotranspiration processes from the trees and plants return
large quantities of water to the atmosphere, promoting cloud formation and rains. Less evaporation means
that more of the sun's energy is used to warm the surface and consequently, the air above, leading to a rise
in temperature. Hence option 1 is correct.
• The increase in CO2, in the atmosphere and the resultant increase in temperatures also lead to floods and
droughts. Because of excessive heat, water evaporates quickly. The soil affected with erosion and
devoid of vegetative covers cannot retain moisture for long. Hence, this causes droughts. Hence option 2
is correct.
• An increasing amount of forest removal also results in a decreasing amount of rainfall.
Decreased rainfall reduces the percolation of water in the underground and consequently lowers the
levels of the underground water table. All this leads to drying up of rivers, streams, lakes and
aquifers. Hence option 3 is not correct.

Q 66.B
• National Wasteland Development Board was established in 1985 under the Ministry of Forests and
Environment mainly to tackle the problem of degradation of lands, restoration of ecology and to meet the
growing demands of fuelwood and fodder at the national level. Hence statement 1 is not correct.
22 www.visionias.in ©Vision IAS
• During the Seventh Five Year Plan, the strategy adopted by the National Wasteland Development Board
emphasised more on tree planting activities rather than Community Participation for wasteland
development.
• In the year 1992, the new Department under the Ministry Of Rural Development (now Ministry of Rural
Areas and Employment) was created and the National Wasteland Development Board was placed under it.
• The Board was reconstituted in August 1992 and was made responsible for the main development of
wastelands in non-forest areas in totality by involving local people at every stage of development.
Hence statement 2 is correct.
• It aims at creating a scenario where the Government acts as a facilitator and the people at the grass-root
level become the real executioner of the programme. Major programme implemented for improving the
productivity of waste & degraded lands keeping in view the poverty, backwardness, gender & equity is
Integrated Wasteland Development Programme.
Q 67.C
• The term Biomagnification means increasing the concentration of various toxic substances along
the food chain. Toxic substances at the level of primary producers get concentrated at each trophic level
as they move up the food chain. Thus representing an inverted pyramid of transfer of toxic
accumulation. Hence statement 1 is correct.
• The phenomenon of concentrated toxic deposition at a higher trophic level is known as bio-accumulation.
• A small amount of toxic constituent which is neither excreted nor metabolised, i.e. it non-
degradable and fat-soluble, gets increased as the food chain moves upward from one trophic level to
the next and the toxic constituents become concentrated. Hence statement 2 is correct.
• An example of the phenomenon of bio-magnification was first noticed in Illinois (USA) where elm trees
were spread with DDT. A large number of birds like robins died near these trees. It was later discovered
that these birds perished due to DDT poisoning. The lethal dose came from the earthworms which they
consumed. Earthworms had concentrated DDT residue by feeding on fallen elm leaves.
Q 68.B
• Recent Context: Centre for Cellular and Molecular Biology (CCMB) has decoded whole-genome
sequences and submitted them to a global
database of SARS-CoV-2 (coronavirus). Of the 2,000 genome sequences submitted to GISAID database
from India, the majority are from CCMB.
• Global Initiative on Sharing All Influenza Data (GISAID) is a global science initiative and primary
source that provides open-access to genomic data of influenza viruses and the novel coronavirus
responsible for COVID-19.
• It is a collaboration involving representatives of Member States, scientists of the Global Influenza
Surveillance and Response System (GISRS) and Global Influenza Programme (GIP) of the WHO, and
experts in licensing intellectual property. Hence statement 3 is correct.
• The GISAID platform was launched on the occasion of the Sixty-first World Health Assembly in May
2008. Created as an alternative to the public domain sharing model, GISAID's sharing mechanism took
into account the concerns of Member States by providing a publicly accessible database designed by the
scientist for scientist, to improve the sharing of influenza data. The data is provided free-of-charge to all
individuals that agreed to identify themselves and agreed to uphold the GISAID sharing mechanism
governed through its Database Access Agreement. Hence statement 1 is correct.
• Germany is the official host of the GISAID platform and database in a public-private partnership with
the administrative arm of GISAID, which ensures oversight of data access compliance matters to protect
the interests of both data providers and the user community. Though WHO provides the technical
oversight of the database and does not host the database. Hence statement 2 is not correct.
• During Covid-19 pandemic, several countries have deposited more than 4,000 sequences of novel
coronavirus and India itself has deposited two.

Q 69.B
• About 75 % of the solar energy reaching the earth is absorbed by the earth’s surface, which increases its
temperature. The rest of the heat radiates back to the atmosphere.
• The atmosphere traps the sun’s heat near the earth’s surface and keeps it warm. This is called the
natural greenhouse effect because it maintains the temperature and makes the earth perfect for
life.
• In a greenhouse, visible light passes through the transparent glass and heats up the soil and the plants. The
warm soil and plants emit infrared radiations. Since glass is opaque to infrared (heat) radiations, it partly
reflects and partly absorbs these radiations.

23 www.visionias.in ©Vision IAS


• Similarly, carbon dioxide molecules also trap heat as they are transparent to sunlight but not to the heat
radiation. If the amount of carbon dioxide crosses the delicate proportion of 0.03 per cent, the natural
greenhouse balance may get disturbed.
• Carbon dioxide is a major contributor to global warming. Besides carbon dioxide, other greenhouse
gases are methane, water vapour, nitrous oxide, CFCs and ozone. Carbon monoxide is not a
greenhouse gas. Hence option (b) is correct.

Q 70.D
• Grassland is an area or a region in which the vegetation is dominated by a nearly continuous cover of
grasses. Grasslands occur in environments conducive to the growth of this plant cover but not to that of
taller plants, particularly trees and shrubs.
• Statement 1 is not correct: Grasslands occupy a vast area of land within continental
interiors. Grasses and variety of herbs constitute the main vegetation of these grasslands. They also
comprise some of the greatest agricultural areas of the world of corn, cattle, and wheat farming.
• These grasslands lie further from the Equator in areas with hot summers and harsh winters. Rainfall is
moderate and occurs in late spring and early summer. The major temperate grasslands are the Veldts of
South Africa, the Puszta of Hungry, the Pampas of Argentina and Uruguay, the steppes of the former the
Soviet Union and the prairies of Central North America.
• Statement 2 is not correct: The grasslands are the most neglected ecosystems in India. They are
'common' lands of the community and although they constitute the most productive systems, they are
controlled by no one. They remain unprotected unless they are notified as Protected Areas under the
Wildlife Protection Act, 1972 or notified as Protected or Reserved forest under the Indian Forest
Act.

Q 71.A
• Green chemistry is a way of thinking and is about utilizing the existing knowledge and principles of
chemistry and other sciences to reduce the adverse impact on the environment. Green chemistry is a
production process that would bring about minimum pollution or deterioration to the environment.
• For example - Plastics being developed from eco-friendly renewable sources, plus some modern plastics
are biodegradable. The combination of innovations reduces our dependence on petroleum products,
protects humans and wildlife from undesirable chemicals in old plastics, and reduces waste and impact on
the environment, Scientists makes food containers from a polymer called polylactic acid, made
using microorganisms to convert cornstarch into a resin. The resulting polymer is used to replace rigid
petroleum-based plastic used in yogurt containers and water bottles. Hence, option (a) is the correct
answer.
• The byproducts generated during a process, if not used gainfully, add to the environmental pollution. Such
processes are not only environmentally unfriendly but also cost-ineffective. The Waste generation and its
disposal both are economically unsound. Utilization of existing knowledge base for reducing the chemical
hazards along with the developmental activities is the foundation of green chemistry.

Q 72.A
• Gir National Park & Wildlife Sanctuary
o Renowned for its majestic Asiatic lions, Gir forest spreads over an area of 1412 sq km, of which
1154 sq km is protected as a wildlife sanctuary and remaining 258 sq km as a national park.
24 www.visionias.in ©Vision IAS
o Gir wildlife sanctuary lies in Talala in Gir Somnath district of Gujarat and is one of the most visited
wildlife sanctuaries in India. It boasts of a thriving lion population, that grew from 359 in 2005 to 523
in 2015.
o The sanctuary is abound in floral and faunal diversity. More than 2000 distinct species of shrubs,
plants, evergreen and semi-evergreen type of trees are present in the sanctuary.
o It is also habitat to various species of mammals, birds, reptiles and insects. Besides Asiatic lions, you
can spot many other carnivorous animals, such as striped hyena, leopard, jungle cat and Indian gray
mongoose.
o The reptile population in the sanctuary includes mugger crocodile, Indian cobra, python and monitor
lizard. Hence the correct answer is option (a)
• The Sundarbans
o It is a mangrove area in the delta formed by the confluence of the Ganges, Brahmaputra and Meghna
Rivers in the Bay of Bengal. It spans from the Hooghly River in India's state of West Bengal to the
Baleswar River in Bangladesh.
o Four protected areas in the Sundarbans are enlisted as UNESCO World Heritage Sites, viz.
Sundarbans National Park, Sundarbans West, Sundarbans South and Sundarbans East Wildlife
Sanctuaries.
o The endangered species that live within the Sundarbans and extinct species that used to be include the
royal Bengal tigers, estuarine crocodile, northern river terrapins (Batagur baska), olive ridley sea
turtles, Gangetic dolphin, ground turtles, hawksbill sea turtles and king crabs (horse shoe).
• Jim Corbett National Park
o It is the oldest national park in India and was established in 1936 as Hailey National Park to protect
the endangered Bengal tiger. It is located in Nainital district and Pauri Garhwal district of Uttarakhand
and was named after Jim Corbett, a well known hunter and naturalist. The park was the first to come
under the Project Tiger initiative.
• Sariska Wildlife Sanctuary
o Nestled in the Aravali Hills over an area of about 800 sq. Km covering the grasslands, dry deciduous
forests, cliffs and rocky landscape, lays the Sariska National Park, now known as The Sariska Tiger
Reserve.
o This area was once hunting preservation of The Maharaja of Alwar. The Reserve is known for its
majestic Royal Bengal Tigers. It was declared as a national park in 1982.
o The protected area is a part of the Aravali Range and the Kathiawar-Gir dry deciduous forests
ecoregion. In 1978, it was given the status of Tiger Reserve under the Project Tiger.
o It is the first tiger reserve to relocate the Tigers (from Ranthambore) successfully. As it lies in the lap
of Aravali Hills, it holds an abundant amount of mineral resources like copper.
Q 73.A
• Statements 1 and 2 are correct: The Central Government can take the following measures for the
protection of the environment and for the prevention, control and abatement of pollution. Under the
Environment (Protection) Act, 1986, the power of the Central Government includes:
o Planning and execution of programmes to prevent, control and abate environmental pollution on an-
all India basis.
o Making rules in respect of all matters pertaining to environmental protection as well as to pollution.
o Setting standards for the emission of pollutants from any source.
o Laying down the standards for environmental quality.
o Laying down procedures and safeguards for the handling of hazardous.
o Assessing manufacturing processes, materials and substances which could cause environmental
pollution.
o Restricting areas in which industries or other establishments are to be set up.
• Statement 3 is not correct: The Forest (Conservation) Act, 1981: This Act was amended in 1988. The
special feature of this Act is that it compels a State Government to take 'prior approval' of the
Government of India, if it de-reserves a reserved forest, that is to say, if the State Government
wants to use forest land for a non-forest purpose.

Q 74.D
• Eutrophication is an enrichment of water by nutrient salts that causes structural changes to the ecosystem
such as: increased production of algae and aquatic plants, depletion of fish species, general deterioration
of water quality and other effects that reduce and preclude use.
• It is caused by human activities because they are responsible for the addition of 80% nitrogen and
75% phosphorous in lake and stream.
25 www.visionias.in ©Vision IAS
• Some phosphorus-rich sources that enrich water bodies with the nutrient include:
o Fertilizers
o Untreated sewage
o Detergents containing phosphorus
o Industrial discharge of waste.
• Among these sources, the primary contributors to eutrophication include agriculture and industrial wastes.

Q 75.D
• Acid rain refers to any precipitation (rain, fog, mist, snow) that is more acidic than normal (pH of less
than 5.6. pH below 7 is acidic).
• Acid rain is caused by atmospheric pollution from acidic gases such as sulphur dioxide and oxides of
nitrogen emitted from the burning of fossil fuels.
• Harmful effects of acid rain:
o chronic bronchitis, pulmonary emphysema and cancer.
o The decrease in the respiration of soil organisms.
o Premature ageing of leaves.
o Prodigious production of lichens on affected trees.
o Decrease the rate of decomposition.
o Acidic medium promotes leaching of heavy metals like aluminium, lead and mercury. Such metals
when percolating into groundwater affect soil microflora/fauna.etc
• Hence option (d) is correct.

Q 76.B
Nitrogen dioxide:
• The irritant red haze in the traffic and congested places is due to oxides of nitrogen. Higher concentrations
of NO2 damage the leaves of plants and retard the rate of photosynthesis.
• Nitrogen dioxide is a lung irritant that can lead to acute respiratory disease in children. It is toxic to
living tissues also. Nitrogen dioxide is also harmful to various textile fibres and metals. Hence option
(b) is correct.

Q 77.D
• The United Nations Conference on the Human Environment (also known as the Stockholm
Conference) was an international conference convened under United Nations auspices held in
Stockholm, Sweden from June 5-16, 1972. It was the UN's first major conference on international
environmental issues, and marked a turning point in the development of international
environmental politics. Hence the correct answer is option (d).
• The Convention on International Civil Aviation, also known as the Chicago Convention, established
the International Civil Aviation Organization (ICAO), a specialized agency of the UN charged with
coordinating international air travel. The Convention establishes rules of airspace, aircraft registration and
safety, security, and sustainability, and details the rights of the signatories in relation to air travel. The
Convention also contains provisions pertaining to taxation.
• The Convention on the Prevention of Marine Pollution by Dumping of Wastes and Other Matter
1972, commonly called the "London Convention" or "LC '72" and also abbreviated as Marine
Dumping, is an agreement to control pollution of the sea by dumping and to encourage regional
agreements supplementary to the Convention. It covers the deliberate disposal at sea of wastes or other
matter from vessels, aircraft, and platforms. It does not cover discharges from land-based sources such as
pipes and outfalls, wastes generated incidental to normal operation of vessels, or placement of materials
for purposes other than mere disposal, providing such disposal is not contrary to aims of the Convention.
• The Vienna Convention for the Protection of the Ozone Layer is a multilateral environmental
agreement signed in 1985 that provided frameworks for international reductions in the production of
chlorofluorocarbons due to their contribution to the destruction of the ozone layer, resulting in an
increased threat of skin cancer. The Vienna Convention provided the framework necessary to create
regulatory measures in the form of the Montreal Protocol. The Kigali Amendment to the Montreal
Protocol is an international agreement to gradually reduce the consumption and production of
hydrofluorocarbons.

26 www.visionias.in ©Vision IAS


Q 78.C
• National Forest Policy, 1988-The basic objectives that should govern the National Forest Policy are:
o Maintenance of environmental stability through preservation and, where necessary, restoration of the
ecological balance that has been adversely disturbed by serious depletion of the forests of the country.
o Conserving the natural heritage of the country by preserving the remaining natural forests with the
vast variety of flora and fauna, which represent the remarkable biological diversity and genetic
resources of the country.
o Meeting the requirements of fuel-wood, fodder, minor forest produce and small timber of the
rural and tribal populations.
o Increasing the productivity of forests to meet essential national needs. And thus increasing
tribal employment. Hence statement 1 is correct.
o Encouraging efficient utilisation of forest produce and maximising substitution of wood.
o Checking soil erosion and denudation in the catchment areas of rivers, lakes, reservoirs in the "interest
of soil and water conservation, for mitigating floods and droughts and for the retardation of siltation
of reservoirs. Checking the extension of sand-dunes in the desert areas of Rajasthan and along
the coastal tracts. Hence statement 2 is correct.
o Increasing substantially the forest/tree cover in the country through massive afforestation and social
forestry programmes, especially on all denuded, degraded and unproductive lands.
o Creating a massive people's movement with the involvement of women, for achieving these objectives
and to minimise pressure on existing forests.

Q 79.C
• The Atomic Energy Act 1962, provide for the development, control and use of atomic energy for
the welfare of the people of India and for other peaceful purposes and for matters connected
therewith.
• It also provides for control over radioactive substances or radiation generating plant in order to —
o prevent radiation hazards;
o secure public safety and safety of persons handling radioactive substances or radiation generating
plant; and
o ensure safe disposal of radioactive wastes;
• It states, that no minerals, concentrates and other materials which contain uranium in its natural state in
excess of such proportion as may be prescribed by notification by the Central government shall be
disposed off except with the previous permission in writing of the Central Government and in accordance
with such terms and conditions as it may impose.
• All offences under this Act shall be cognizable under the Code of Criminal Procedure, 1898. Non
compliance shall be punishable with imprisonment for a term which may extend to
o one year (for an individual) and
o five years for a company, or with fine, or with both.
• Hence both the statements are correct.

Q 80.B
• In wildlife sanctuaries, rare and endangered species are encouraged to breed in human controlled
environments with restricted settings. This is called as captive breeding.
• This is a successful technique to increase the populations of rare and endangered species of animals.
When the populations of such animals are increased through captive breeding in the captivity the animals
are safely released into the wild.
• The technique of captive breeding has following advantages –
o It helps to increase the populations of rare and endangered species of animals and to save these
animals from extinction.
o The birth rates, hatching rates and survival rates of captivity –produced animals remains much higher
than in case of higher bleedings.
o This technique helps us to raise the population of wild animals up to the desired levels.
o This technique is useful in developing desired characters or traits in organisms.
• However, this technique does not ensure the protection of the natural habitat of species.
• Hence option (b) is correct.

27 www.visionias.in ©Vision IAS


Q 81.D
• Bio-piracy refers to the illegal collection and patenting of the biological materials originally
belonging to some other community, state or nation. In other words, biopiracy is defined as a
situation where indigenous knowledge of nature, originating with indigenous people, is used by
others for profit, without permission from and with little or no compensation or recognition to the
indigenous people themselves.
• Bio-piracy relates to:
o illegal unauthorised use of biological material like plants, animals or their products.
o illegal and unauthorised use of traditional knowledge of some other society or community about the
application of certain biological products.
o unlawful sharing in the benefits by the patent holder and deprivation of indigenous communities or
people who are the real owner of the biological material/knowledge used for making benefits.
• Developed countries are exploiting developing countries genetic resources and indigenous communities
traditional knowledge in the name of patents on the inventions derived from those genetic resources.

Q 82.B
• The World Water Council is an non-profit international multi-stakeholder platform. It was
established in 1996 on the initiative of renowned water specialists and international organisations
and regulated according to the French law of July 1, 1901, known as "Association Loi de 1901", in
response to an increasing concern about world water issues from the global community. Hence statement
1 is not correct.
• The Council focuses on the political dimensions of water security, adaptation and sustainability. The
World Water Council's mission is to promote awareness, build political commitment and trigger action on
critical water issues at all levels, including the highest decision-making level, to facilitate the efficient
conservation, protection, development, planning, management and use of water in all its dimensions on an
environmentally sustainable basis for the benefit of all life on earth. Hence statement 2 is correct.
• By providing a platform to encourage debates and exchanges of experience, the Council aims to reach a
common strategic vision on water resources and water services management amongst all stakeholders in
the water community. In the process, the Council also catalyses initiatives and activities, whose results
converge toward its flagship product, the World Water Forum.

Q 83.A
• Wastes can be classified into following categories:
o Biodegradable wastes
o Non-biodegradable wastes
o Toxic wastes
o Non-toxic wastes
o Bio-medical wastes
o E-wastes
• Biodegradable wastes: Wastes that can be degraded or broken down through microbial activities of
fungii and bacteria are called as biodegradable wastes.
• Biodegardable wastes can be classified as:
o Simple Biodegradable waste: These wastes are easily broken down by natural processes of
decomposition. Leaves, vegetable peels, plant remains, animal remains, faecal remains, dead plants
and animals and waste water.
o Complex Biodegradable waste: They are resistent to natural processes of decomposition. However,
over a long period of time they are decomposed. For example glass bottle takes a million years to
decompose, leather shoes take 30-40 years, tin cans 50-100 years, etc.
• Non-biodegradable wastes: Wastes which cannot be degraded or broken down through microbial
activities are called as non-biodegradable wastes. Such wastes include crude petroleum, plastics,
styrofoam products, cans, glasses, polymer, synthetic pesticides, radioactive fallout, some industrial
effluents and metals like lead. Hence the correct option is (a)
• Toxic Wastes: those which pose a serious threat to human health and environment. For example: certain
industrial chemicals and medical waste.
• Non-toxic Wastes: solid waste from domestic activities such as food leftover, fruits, etc.
• Bio-medical wastes: waste generated from the activities from hospital like syringes, body fluids, etc.
• E-Wastes: discarded electronic equipments like mobiles, computers, etc.

28 www.visionias.in ©Vision IAS


Q 84.C
• Korvai is an ancient and complex technique of weaving, practiced in Kanchipuram, Tamil
Nadu. The word ‘korvai’ in Tamil means ‘in sync’. Korvai is used to interlace the borders with the
body of the sari. The body of the sari is in a different colour contrasting with the border which is in one
solid colour where the weft of the body colour does not interlock with the warp threads of the border
making it a double shade. Hence option (c) is the correct answer.
• It is labour intensive and requires two weavers, one to operate the threads for the central portion and
another to operate the border colours. Korvai is also distinguished by its motifs, often inspired by
the beautiful carvings found in the temples of Kanchipuram. The Korvai-weaving technique is a
timeless classic, passed on from one generation of weavers to another.
• In the wake of the power loom revolution, this technique seemed to be replaced by easier and cheaper
substitutes. Today, however, there is a gradual revival of this art with the help of the Crafts Council of
India and enthusiastic revivalists.
Q 85.A
• Keystone species: Some species of wildlife are known to play more significant roles in particular
ecosystems than other species. Such species are called as keystone species. The loss of such species
becomes seriously destructive to ecosystems. Some examples of keystone species include the bison,
prairie dog, and otter. Hence option (a) is correct.
• Ruderal species:It is a plant species that is first to colonize disturbed lands. The disturbance may be
natural – for example, wildfires or avalanches – or a consequence of human activity, such as construction
(of roads, of buildings, mining, etc.) or agriculture (abandoned fields, irrigation, etc.).
• Indicator species: organisms often a microorganism or a plant—that serves as a measure of the
environmental conditions that exist in a given locale. For example, greasewood indicates saline soil;
mosses often indicate acid soil. Tubifex worms indicate oxygen-poor and stagnant water unfit to drink.
• Invasive species: They are not native to an ecosystem and causes harm to the environment, the economy,
or even human health. Species that grow and reproduce quickly, and spread aggressively.
Q 86.B
• The Convention on the Prevention of Marine Pollution by Dumping of Wastes and Other Matter 1972,
commonly called the “London Convention” or “LC ’72" and also abbreviated as Marine Dumping, is an
agreement to control pollution of the sea by dumping and to encourage regional agreements
supplementary to the Convention.
• The London Dumping Convention was called for by the United Nations Conference on the Human
Environment (June 1972, Stockholm); the treaty was drafted at the Intergovernmental Conference on the
Convention on the Dumping of Wastes at Sea (November 13, 1972, London) and it was opened for signature
on December 29, 1972.
• It entered into force on August 30, 1975 when 15 nations ratified International Administration of the
Convention functions through Consultative Meetings held at International Maritime Organization
(IMO) headquarters in London.
• It covers the deliberate disposal at sea of wastes or other matter from vessels, aircraft, and
platforms. It does not cover discharges from land-based sources such as pipes and outfalls, wastes
generated incidental to normal operation of vessels, or placement of materials for purposes other than
mere disposal, providing such disposal is not contrary to aims of the Convention.
• The London Protocol -1996 makes it binding on the parties to follow the provisions of the convention. As of
2005, there were 81 Parties to the Convention. India is not a party to this convention.
• The main objective of the London Convention is –
o To prevent indiscriminate disposal at sea of wastes that could be liable for creating hazards to human
health; Harming living resources and marine life; Damaging amenities; or interfering with other legitimate
uses of the sea.
o The 1972 Convention extends its scope over “all marine waters other than the internal waters” of the States
and prohibits the dumping of certain hazardous materials. It further requires a prior special permit for the
dumping of a number of other identified materials and a prior general permit for other wastes or
matter. Hence the correct answer is option (b).
Q 87.A
• A harmful way in which acids can enter the lakes is spring acid shock. When the snow melts in spring
rapidly due to a sudden temperature change, the acids and chemicals in the snow are released into the
soils. The melted snow then runs off to streams and rivers, and gradually make their way into the
lakes. The introduction of these acids and chemicals into the lakes causes a sudden drastic change in
the pH of the lakes - hence the term "spring shock or spring acid shock".

29 www.visionias.in ©Vision IAS


• The aquatic ecosystem has no time to adjust to the sudden change. In addition, springtime is an especially
vulnerable time for many aquatic species since this is the time for reproduction for amphibians, fish and
insects. Many of these species lay their eggs in the water to hatch.
• The sudden pH change is dangerous because the acids can cause serious deformities in their young or
even annihilate the whole species since the young of many of such species spend a significant part of their
life cycle in the water. Hence option (a) is the correct answer.

Q 88.C
• Statement 1 is correct: The carrying capacity of an environment is the maximum population size of
a biological species that can be sustained in that specific environment, given the food, habitat,
water, and other resources available.
• Statement 2 is correct: The carrying capacity is different for each species in a habitat because of
that species’ particular food, shelter, and social requirements. Disease, competition, predator-prey
interaction, resource use and the number of populations in an ecosystem all affect carrying capacity. It
does not relate to the diversity of species. Important direct drivers affecting biodiversity are habitat
change, climate change, invasive species, overexploitation, and pollution.
• Populations grow through births and immigration and decline through deaths and emigration. When
resources are unlimited, the growth is usually exponential but when resources become progressively
limiting, the growth pattern turns logistic. The intrinsic rate of natural increase (r) is a measure of the
inherent potential of a population to grow. The ‘intrinsic rate of natural increase’ is a very important
parameter chosen for assessing impacts of any biotic or abiotic factor on population growth.
Q 89.A
• Recent Context: Union Minister for Finance and Corporate Affairs launched the NIP i.e. National
Infrastructure Pipeline Online Dashboard. The online dashboard is predicted to be a one-stop destination
for all stakeholders looking for information that is related to infrastructure projects in the New India. IIG
i.e. India Investment Grid will be hosting the dashboard (www.indiainvestmentgrid.gov.in).
• Statement 1 is correct: National Infrastructure Pipeline is the investment plan unveiled by the Central
Government for enhancing infrastructure in identified sectors for a period of five years from 2020-25.
• Finance Minister announced Rs 102 lakh crore ($1.4 trillion) National Infrastructure Pipeline to spend in
the infrastructure sector over a five-year period (2019-25). The plan will help India to reach $5 trillion
economy by 2025.
• Statement 2 is not correct: The funding of the National Infrastructure Pipeline will be jointly made by
the Centre, states and the private sector in the proportion of 39:39:22 (39 % each by the centre and states
and 22% by the private sector) The infrastructure plan was proposed by the Taskforce on National
Infrastructure Pipeline for 2019-2025.
Q 90.A
• The energy conservation helps in increasing national security, personal security, financial capital, human
comfort and environmental values.
• Some examples of the methods of energy conservation are :
o Better insulation of buildings.
o Recovery of waste heat produced during many industrial processes and its better use.
o Encourage travel by bus and train rather than by private cars.
o Encouraging the use of LEDs because these lights produce more light per unit of electricity than
other lights do
o Lowering of thermostats in winter and raising them during summer.
o Turning off unnecessary lights.
o Recycling of paper and metals.
• Hence option (a) is correct.

Q 91.C
• The kangaroo rat is a very small rat found in arid regions of the Western and South-Western US. One
behaviour that makes kangaroo rats so unique is that they never drink water.
• In the absence of an external source of water, the kangaroo rat in North American deserts is
capable of meeting all its water requirements through its internal fat oxidation (in which water is a
by-product). It also has the ability to concentrate its urine so that minimal volume of water is used to
remove excretory products. Hence options 1 and 2 are correct.
• Kangaroo rats are mostly seed-eaters, eating mostly mesquite beans and grass seeds. Occasionally
the Kangaroo rat can be seen eating small insects. Kangaroo rats will forage and collect seeds at night,
storing seeds and beans in their cheek pouches.
30 www.visionias.in ©Vision IAS
Q 92.C
• Recent Context: In light of recent developments in the world related to pandemic COVID-19, it was
decided that the 44th session of the World Heritage Commi8ee, initially scheduled from June 29 - July 9,
2020 will be postponed to a later date. The Ramappa temple was nominated from Indian side for the
Unesco World Heritage Site.
• The Ramalingeswara Temple which is popularly known as the Ramappa temple is an amazing piece
of art that stands as a testimony of the royal Kakatiyas. It is located in Telengana. Hence option (c) is
the correct answer.
• The temple got its name Ramappa because of its chief sculptor Ramappa. It's probably the only temple
in the country to be known by the name of its sculptor.
• The medieval Deccan Ramappa Temple which dates back to 1213 AD, was built by the patronage of the
Kakatiya ruler Kakati Ganapathi Deva under the authority of his Chief Commander Rudra Samani at the
place known as Ranakude in the Atukuru province.
• A striking feature of this temple is that it is built with bricks that are so light that they can easily float on
water. The temple which is situated in a valley took nearly 40 years to be completed. Elaborate carvings line
the walls and also cover the pillars and ceilings of the temple.
• The Ramappa temple finds its hold on a 6 feet high platform on a cruciform plan. The temple’s chamber is
crowned with a shikharam and is surrounded by pradakshinapatha. At the entrance of the temple, we can find
a Nandi mandapam. An imposing Nandi vigraham stands on it.

Q 93.D
• Ozone is one of the three allotropes of oxygen, an element in gaseous form. It is triatomic and less stable
than oxygen. Its chemical formula is 03. Ozone in the stratosphere is very important to life. It is formed
by the action of the ultraviolet light from the sun on molecules of oxygen.
• A large number of ozone molecules assemble around the earth to form the Ozone Layer which extends
from 12 to 45 km above the earth surface. On an average it is about 230 Dobson units (DU) in
thickness. DU is the unit which measures thickness of the ozone layer.
• Ozone absorbs ultraviolet radiations so that much of it is never allowed to reach to the earth surface.
The protective umbrella of ozone layer in the stratosphere protects the earth Aom harmful ultraviolet
radiations. Ozone plays an important role in the biology and climatology on the earth’s environment. It filters
out all the radiations that remain below 3000â‹. Radiations below this wavelength are biologically harmful.
Hence any depletion of ozone layer is sure to exert catastrophic impacts on life in the biosphere.
• Substances that cause depletion of Ozone Layer: Chlorofluorocarbons, nitrous oxides (N2O), carbon
tetrachloride (CCl4), methyl bromide (a soil fumigant and insecticide), aircraft emissions, n- propyl
bromide and Halon- 1202 are major agents that cause depletion of ozone layer. Hence, these are called as
Ozone Depleting Substances (ODS). Hence the correct answer is option (d).
• In March 1985, 28 countries of the world agreed on Vienna Convention for the protection of the ozone
layer. In September 1987, different countries of the world adopted Montreal Protocol on substances that
deplete ozone layer. Kigali Agreement to reduce the manufacture and use of Hydrofluorocarbon (HFC).

Q 94.D
• The biotic factors refer to all the living beings present in an ecosystem and abiotic factor is a non- living
part of an ecosystem that shapes its environment.
• Temperature is the most important ecologically relevant environmental factor. It ranges from subzero
levels in polar areas and high altitudes to +500C in tropical deserts in summer. There are, however,
unique habitats such as thermal springs and deep-sea hydrothermal vents where average temperatures
exceed 100 Degree C. The significance of temperature to living organisms is that it affects the kinetics of
enzymes and through it the metabolic activity and other physiological functions of the organism.
• Water is another the most important factor inñuencing the life of organisms. In fact, life on earth
originated in water and is unsustainable without water. Its availability is so limited in deserts that only
special adaptations make it possible for organisms to live there. The productivity and distribution of plants is
also heavily dependent on water.
• Since plants produce food through photosynthesis, a process which is only possible when sunlight is
available as a source of energy. Many species of small plants (herbs and shrubs) growing in forests are
adapted to photosynthesise optimally under very low light conditions because they are constantly
overshadowed by tall, canopied trees. Many plants are also dependent on sunlight to meet their
photoperiodic requirement for flowering.

31 www.visionias.in i2Vision IAS


• The nature and properties of soil in different places vary; it is dependent on the climate, the weathering
process, whether soil is transported or sedimentary and how soil development occurred. Various
characteristics of the soil such as soil composition, grain size and aggregation determine the percolation and
water holding capacity of the soils. These characteristics along with parameters such as pH, mineral
composition and topography determine to a large extent the vegetation in any area.

Q 95.B
• Based on the nature of the habitat — whether it is water (or very wet areas) or it is on very dry areas — a
succession of plants is called hydrarch or xerarch, respectively. Hydrarch succession takes place in wet areas
and the successional series progress from hydric to the mesic conditions. It takes years for the climax
to be reached. All succession whether taking place in water or on land, proceeds to a similar climax
community — the mesic (In ecology, a mesic habitat is a type of habitat with a moderate or well-
balanced supply of moisture, e.g., a mesic forest, a temperate hardwood forest, or dry-mesic prairie).
• In primary succession in water, the pioneers are the small phytoplankton, which is replaced with time
by rooted-submerged plants, rooted-floating angiospems followed by free-floating plants, then reed
swamp, marsh-meadow, scrub and finally the trees. The climax again would be a forest. With time the
water body is converted into the land.

I ’1 i i i › i1. i i th I ‹ › i i lt‹ r'i I s- i '. i ii i › t. i i


\:‹l I‹t I

›, ii › i t\‹ ”I ‹’‹ I ) › l ‹i t ii ). i ›'


lt›)

I tfI

Q 96.D

• Statement 1 is not correct: According to the International Union for Conservation of Nature and Natural
Resources (IUCN) (2004), the total number of plant and animal species described so far is slightly more than
1.5 million. More than 70 per cent of all the species recorded are animals, while plants (including
algae, fungi, bryophytes, gymnosperms and angiosperms) comprise no more than 22 per cent of the
total.
• Statement 2 is not correct: Among animals, insects are the most species-rich taxonomic group, making
up more than 70 per cent of the total. That means, out of every 10 animals on this planet, 7 areinsects.
Insect biodiversity accounts for a large proportion of all biodiversity on the planet over half of the
estimated 1.5 million organism species described are classified as insects.
32 www.visionias.in i2Vision IAS
Q 97.A
• Recent Context: India has recently has contributed $15.46 million to the India-U.N. Development
Partnership Fund. The tranche of $15.46 million includes $6 million to the overall fund, in which all the
developing countries are eligible for partnership, and $9.46 million dedicated to the Commonwealth
countries.
• The India-UN Development Partnership Fund is a dedicated facility within the United Nations
Fund for South-South Cooperation established in 2017. It is supported and led by the Government of
the Republic of India, managed by the United Nations Office for South-South Cooperation
(UNOSSC), and implemented in collaboration with the United Nations system. Hence, statement 2
is not correct.
• The India-UN Development Partnership Fund supports Southern-owned and led, demand-driven,
and transformational sustainable development projects across the developing world, with a focus on
least developed countries and small island developing states. Hence statement 1 is correct.
• The Fund project portfolio aims to contribute to the efforts of developing countries towards the realization
of the 2030 Agenda for Sustainable Development.
• The Fund responds directly to the national priorities and development objectives of partner countries,
contributing financial resources and technical knowledge to support partner governments in achieving the
Sustainable Development Goals
• South-South cooperation refers to the technical cooperation among developing countries in the Global
South. It is a tool used by the states, international organizations, academics, civil society and the private
sector to collaborate and share knowledge, skills and successful initiatives in specific areas such as
agricultural development, human rights, urbanization, health, climate change etc.

Q 98.B
• The main reason for ozone layer depletion is believed to be the release of chlorofluorocarbons
compounds (CFCs), also known as freons.
• These compounds are nonreactive Hence statement 1 is not correct,
• These compounds are non-flammable, non-toxic organic molecules and therefore used in
refrigerators, air conditioners, in the production of plastic foam and by the electronic industry for
cleaning computer parts etc. Hence statements 2 and 3 are correct.
• Once CFCs are released in the atmosphere, they mix with the normal atmospheric gases and eventually
reach the stratosphere. In the stratosphere, they get broken down by powerful UV radiations, releasing
chlorine free radical.
• The chlorine radicals then react with stratospheric ozone to form chlorine monoxide radicals and
molecular oxygen.
• The reaction of chlorine monoxide radical with atomic oxygen produces more chlorine radicals.
• The chlorine radicals are continuously regenerated and cause the breakdown of ozone. Thus, CFCs are
transporting agents for continuously generating chlorine radicals into the stratosphere and damaging the
ozone layer.

Q 99.A
• Statement 1 is not correct: Except for the deep sea hydro-thermal ecosystem, the sun is the only
source of energy for all ecosystems on Earth. Hydrothermal vents are home to dozens of previously
unknown species. Huge red-tipped tube worms, ghostly fish, strange shrimp with eyes on their backs, and
other unique species thrive in these extreme deep-ocean ecosystems found near undersea volcanic
chains. Instead of sunlight, vent life relies on hydrogen sulfide - more commonly known as rotten
egg gas and toxic to most land-based life. In a process called chemosynthesis, specialized bacteria
create energy from the hydrogen sulfide present in the mineral-rich water pouring out of the vents.
• Statement 2 is correct: Photosynthetically active radiation, often abbreviated PAR, designates the
spectral range of solar radiation from 400 to 700 nanometers that photosynthetic organisms are able to use
in the process of photosynthesis. Of the incident solar radiation, less than 50 per cent of it is
photosynthetically active radiation (PAR). We know that plants and photosynthetic bacteria
(autotrophs), fix Sun’s radiant energy to make food from simple inorganic materials.
• Statement 3 is not correct: Plants capture only 2-10 per cent of the PAR and this small amount of
energy sustains the entire living world. About 1-5 % of incident solar energy or 2-10 % of PAR is
captured by the photosynthetic organisms for the synthesis of organic matter (Gross primary productivity).
Roughly 20 % of it is consumed in respiration so that the net capture of energy (net primary productivity)
is 0.8-4 % of incident radiation or 1.6-8% of PAR.

33 www.visionias.in ©Vision IAS


Q 100.C
• Statement 1 is correct: Green Manuring is an agricultural and horticultural practise of growing a
plant crop specifically in order to plough or dig into the soil where it improves the structure and
releases nutrients as it decomposes. In agriculture, green manure is also created by leaving uprooted or
sown crop parts to wither on a field so that they serve as a mulch and soil amendment.
• Statement 2 is correct: Green manure is a type of cover crop grown to add nutrients and organic matter
to the soil. Typically, a green manure crop is grown for a specific period. Leguminous plants are largely
used as green manure. Leguminous green manures such as clover contain nitrogen-fixing bacteria
in root nodules that fix atmospheric nitrogen in a form that plants can use.
• Green manures usually perform multiple functions that include soil improvement and soil protection.
They increase the percentage of organic matter (biomass) in the soil, thereby improving water retention,
aeration and other soil characteristics. Examples of green manure crops include grass mixtures and
legume plants. Some of the most commonly used are Annual ryegrass.

Upsc 4 EveryOne is initiative to provide watermarks and bookmarks free


pdfs to you. Share and Subscribe our telegram channel
@Upsc_4_EveryOne1

https://t.me/Upsc_4_EveryOne1

34 www.visionias.in ©Vision IAS

You might also like